maternal child test 2

अब Quizwiz के साथ अपने होमवर्क और परीक्षाओं को एस करें!

Which statement best describes a child who is abused by the parent(s)? a. Unintentionally contributes to the abusing situation b. Belongs to a low socioeconomic population c. Is healthier than the nonabused siblings d. Abuses siblings in the same way as child is abused by the parent(s)

ANS: A A child's temperament, position in the family, additional physical needs, activity level, or degree of sensitivity to parental needs unintentionally contributes to the abusing situation. Socioeconomic status is an environmentalcharacteristic. This child is less likely to be abused than one who is premature, disabled, or very young. The abused child does not in turn abuse his or her siblings.

A young child with leukemia has anorexia and severe stomatitis. The nurse should suggest that the parents try which intervention? a. Relax any eating pressures. b. Firmly insist that child eat normally. c. Begin gavage feedings to supplement diet. d. Serve foods that are either hot or cold.

ANS: A A multifaceted approach is necessary for children with severe stomatitis and anorexia. First, the parents should relax eating pressures rather than insisting the child eat normally. The nurse should suggest that the parents try soft, bland foods rather than hot or cold foods; normal saline or bicarbonate mouthwashes; and local anesthetics. The stomatitis is a temporary condition; gavage feedings are not necessary. The child can resume good food habits as soon as the condition resolves.

Preschoolers' fears can best be dealt with by which intervention? a. Actively involving them in finding practical methods to deal with the frightening experience b. Forcing them to confront the frightening object or experience in the presence of their parents c. Using logical persuasion to explain away their fears and help them recognize how unrealistic the fears are d. Ridiculing their fears so they understand that there is no need to be afraid

ANS: A Actively involving the child in finding practical methods to deal with the frightening experience is the best way to deal with fears. Forcing a child to confront fears may make the child more afraid. Preconceptual thought prevents logical understanding. Ridiculing fears does not make them go away.

Which activity is most appropriate for developing fine motor skills in the school-age child? a. Drawing b. Singing c. Soccer d. Swimming

ANS: A Activities such as drawing, building models, and playing a musical instrument increase the school-age child's fine motor skills. Singing is an appropriate activity for the school-age child, but it does not increase fine motor skills. The school-age child needs to participate in group activities to increase both gross motor skills and social skills, but group activities do not increase fine motor skills. Swimming is an activity that also increases gross motor skills.

Which aspect of cognition develops during adolescence? a. Capability to use a future time perspective b. Ability to place things in a sensible and logical order c. Ability to see things from the point of view of another d. Progress from making judgments based on what they see to making judgments based on what they reason

ANS: A Adolescents are no longer restricted to the real and actual. They also are concerned with the possible; they think beyond the present. During concrete operations (between ages 7 and 11 years), children exhibit the ability to place things in a sensible and logical order, the ability to see things from another's point of view, and the ability to make judgments based on what they reason rather than just what they see.

A child has an evulsed (knocked-out) tooth. In which medium should the nurse instruct the parents to place the tooth for transport to the dentist? a. Cold milk b. Cold water c. Warm salt water d. A dry, clean jar

ANS: A An evulsed tooth should be placed in a suitable medium for transport, either cold milk or saliva (under the child's or parent's tongue). Cold milk is a more suitable medium for transport than cold water, warm salt water, or a dry, clean jar.

An 18-month-old child is seen in the clinic with AOM. Trimethoprim-sulfamethoxazole (Bactrim) is prescribed. Which statement made by the parent indicates a correct understanding of the instructions? a. "I should administer all the prescribed medication." b. "I should continue medication until the symptoms subside." c. "I will immediately stop giving medication if I notice a change in hearing." d. "I will stop giving medication if fever is still present in 24 hours."

ANS: A Antibiotics should be given for their full course to prevent recurrence of infection with resistant bacteria. Symptoms may subside before the full course is given. Hearing loss is a complication of AOM. Antibiotics should continue to be given. Medication may take 24 to 48 hours to make symptoms subside. It should be continued.

What is the nurse's first action when planning to teach the parents of an infant with a congenital heart defect (CHD)? a. Assess the parents' anxiety level and readiness to learn. b. Gather literature for the parents. c. Secure a quiet place for teaching. d. Discuss the plan with the nursing team.

ANS: A Any effort to organize the right environment, plan, or literature is of no use if the parents are not ready to learn or have high anxiety. Decreasing their level of anxiety is often needed before new information can be processed. A baseline assessment of prior knowledge should be taken into consideration before developing any teaching plan. Locating a quiet place for meeting with parents is appropriate; however, an assessment should be done before any teaching is done. Discussing a teaching plan with the nursing team is appropriate after an assessment of the parents' knowledge and readiness.

In which condition are all the formed elements of the blood simultaneously depressed? a. Aplastic anemia c. Thalassemia major b. Sickle cell anemia d. Iron deficiency anemia

ANS: A Aplastic anemia refers to a bone marrow-failure condition in which the formed elements of the blood are simultaneously depressed. Sickle cell anemia is a hemoglobinopathy in which normal adult hemoglobin is partly or completely replaced by abnormal sickle hemoglobin. Thalassemia major is a group of blood disorders characterized by deficiency in the production rate of specific hemoglobin globin chains. Iron deficiency anemia results in a decreased amount of circulating red cells.

The nurse is planning care for an adolescent with acquired immunodeficiency syndrome. The priority nursing goal is to: a. Prevent infection. c. Restore immunologic defenses. b. Prevent secondary cancers. d. Identify source of infection

ANS: A As a result of the immunocompromise that is associated with human immunodeficiency virus infection, the prevention of infection is paramount. Although certain precautions are justified in limiting exposure to infection, these must be balanced with the concern for the child's normal developmental needs. Restoring immunologic defenses is not currently possible. Current drug therapy is affecting the disease progression; although not a cure, these drugs can suppress viral replication, preventing further deterioration. Case finding is not a priority nursing goal.

The nurse is assessing a child post-cardiac catheterization. Which complication might the nurse anticipate? a. Cardiac arrhythmia c. Congestive heart failure b. Hypostatic pneumonia d. Rapidly increasing blood pressure

ANS: A Because a catheter is introduced into the heart, a risk exists of catheter-induced arrhythmias occurring during the procedure. These are usually transient. Hypostatic pneumonia, congestive heart failure, and rapidly increasing blood pressure are not risks usually associated with cardiac catheterization.

A school-age child has had an upper respiratory tract infection for several days and then began having a persistent dry, hacking cough that was worse at night. The cough has become productive in the past 24 hours. This is most suggestive of: a. Bronchitis. c. Viral-induced asthma. b. Bronchiolitis. d. Acute spasmodic laryngitis.

ANS: A Bronchitis is characterized by these symptoms and occurs in children older than 6 years. Bronchiolitis is rare in children older than 2 years. Asthma is a chronic inflammation of the airways that may be exacerbated by a virus. Acute spasmodic laryngitis occurs in children between 3 months and 3 years.

A child with cystic fibrosis (CF) receives aerosolized bronchodilator medication. When should this medication be administered? a. Before chest physiotherapy (CPT) c. Before receiving 100% oxygen b. After CPT d. After receiving 100% oxygen

ANS: A Bronchodilators should be given before CPT to open bronchi and make expectoration easier. Aerosolized bronchodilator medications are not helpful when used after CPT. Oxygen administration is necessary only in acute episodes with caution because of chronic carbon dioxide retention.

The ability to mentally understand that 1 + 3 = 4 and 4 - 3 = 1 occurs in which stage of cognitive development? a. Concrete operations stage b. Formal operations stage c. Intuitive thought stage d. Preoperations stage

ANS: A By 7 to 8 years of age, the child is able to retrace a process (reversibility) and has the skills necessary for solving mathematical problems. This stage is called concrete operations. The formal operations stage deals with abstract reasoning and does not occur until adolescence. Thinking in the intuitive stage is based on immediate perceptions. A child in this stage often solves problems by random guessing. In preoperational thinking, the child is usually able to add 1 + 3 = 4 but is unable to retrace the process.

Which statement expresses accurately the genetic implications of cystic fibrosis (CF)? a. If it is present in a child, both parents are carriers of this defective gene. b. It is inherited as an autosomal dominant trait. c. It is a genetic defect found primarily in non-Caucasian population groups. d. There is a 50% chance that siblings of an affected child also will be affected.

ANS: A CF is an autosomal recessive gene inherited from both parents and is found primarily in Caucasian populations. An autosomal recessive inheritance pattern means that there is a 25% chance that a sibling will be infected but a 50% chance a sibling will be a carrier.

Which drug is an angiotensin-converting enzyme (ACE) inhibitor? a. Captopril (Capoten) c. Spironolactone (Aldactone) b. Furosemide (Lasix) d. Chlorothiazide (Diuril)

ANS: A Capoten is an ACE inhibitor. Lasix is a loop diuretic. Aldactone blocks the action of aldosterone. Diuril works on the distal tubules.

An 8-year-old girl tells the nurse that she has cancer because God is punishing her for "being bad." She shares her concern that, if she dies, she will go to hell. The nurse should interpret this as being: a. A belief common at this age. b. A belief that forms the basis for most religions. c. Suggestive of excessive family pressure. d. Suggestive of a failure to develop a conscience.

ANS: A Children at this age may view illness or injury as a punishment for a real or imagined mystique. The belief in divine punishment is common at this age.

Which statement best explains why iron deficiency anemia is common during toddlerhood? a. Milk is a poor source of iron. b. Iron cannot be stored during fetal development. c. Fetal iron stores are depleted by age 1 month. d. Dietary iron cannot be started until age 12 months.

ANS: A Children between the ages of 12 and 36 months are at risk for anemia because cow's milk is a major component of their diet, and it is a poor source of iron. Iron is stored during fetal development, but the amount stored depends on maternal iron stores. Fetal iron stores are usually depleted by age 5 to 6 months. Dietary iron can be introduced by breastfeeding, iron-fortified formula, and cereals during the first 12 months of life.

In providing nourishment for a child with cystic fibrosis (CF), which factor should the nurse keep in mind? a. Diet should be high in carbohydrates and protein. b. Diet should be high in easily digested carbohydrates and fats. c. Most fruits and vegetables are not well tolerated. d. Fats and proteins must be greatly curtailed.

ANS: A Children with CF require a well-balanced, high-protein, high-calorie diet because of impaired intestinal absorption. Enzyme supplementation helps digest foods; other modifications are not necessary. A well-balanced diet containing fruits and vegetables is important. Fats and proteins are a necessary part of a well-balanced diet.

A child has a chronic, nonproductive cough and diffuse wheezing during the expiratory phase of respiration. This suggests: a. Asthma. c. Bronchiolitis. b. Pneumonia. d. Foreign body in the trachea.

ANS: A Children with asthma usually have these chronic symptoms. Pneumonia appears with an acute onset and fever and general malaise. Bronchiolitis is an acute condition caused by respiratory syncytial virus. Foreign body in the trachea will manifest with acute respiratory distress or failure and maybe stridor.

Which statement is true about smoking in adolescence? a. Smoking is related to other high-risk behaviors. b. Smoking is more common among athletes. c. Smoking is less common when the adolescent's parent(s) smokes. d. Smoking among adolescents is becoming more prevalent.

ANS: A Cigarettes are considered a gateway drug. Teenagers who smoke are 11.4 times more likely to use an illicit drug. Teens who refrain from smoking often have a desire to succeed in athletics. If a parent smokes, it is more likely that the teen will smoke. Cigarette smoking has declined among all groups since the 1990s.

A nurse is interpreting the results of a tuberculin skin test (TST) on an adolescent who is human immunodeficiency virus (HIV) positive. Which induration size indicates a positive result for this child 48 to 72 hours after the test? a. 5 mm c. 15 mm b. 10 mm d. 20 mm

ANS: A Clinical evidence of a positive TST in children receiving immunosuppressive therapy, including immunosuppressive doses of steroids, or who have immunosuppressive conditions, including HIV infection, is an induration of 5 mm. Children younger than 4 years of age (a) with other medical risk conditions, including Hodgkin's disease, lymphoma, diabetes mellitus, chronic renal failure, or malnutrition; (b) born or whose parents were born in high-prevalence tuberculosis (TB) regions of the world; (c) frequently exposed to adults who are HIV infected, homeless, users of illicit drugs, residents of nursing homes, incarcerated or institutionalized, or migrant farm workers; and (d) who travel to high-prevalence TB regions of the world are positive when the induration is 10 mm. Children 4 years of age or older without any risk factors are positive when the induration is 20 mm.

According to Piaget, the adolescent is in the fourth stage of cognitive development, or period of: a. Formal operations. c. Conventional thought. b. Concrete operations. d. Postconventional thought.

ANS: A Cognitive thinking culminates with capacity for abstract thinking. This stage, the period of formal operations, is Piaget's fourth and last stage. The concrete operations stage usually develops between ages 7 and 11 years. Conventional and postconventional thought refer to Kohlberg's stages of moral development.

A beneficial effect of administering digoxin (Lanoxin) is that it: a. Decreases edema. c. Increases heart size. b. Decreases cardiac output. d. Increases venous pressure.

ANS: A Digoxin has a rapid onset and is useful in increasing cardiac output, decreasing venous pressure, and as a result decreasing edema. Heart size is decreased by digoxin

A possible cause of acquired aplastic anemia in children is: a. Drugs. c. Deficient diet. b. Injury. d. Congenital defect.

ANS: A Drugs such as chemotherapeutic agents and several antibiotics such as chloramphenicol can cause aplastic anemia. Fanconi syndrome is a primary form of the disorder, which is congenital/present-at-birth and not acquired after birth. Injury, deficient diet, and congenital defect are not causative agents in acquired aplastic anemia.

A nurse is charting that a hospitalized child has labored breathing. Which describes labored breathing? a. Dyspnea c. Hypopnea b. Tachypnea d. Orthopnea

ANS: A Dyspnea is labored breathing. Tachypnea is rapid breathing. Hypopnea is breathing that is too shallow. Orthopnea is difficulty breathing except in upright position.

An infant's parents ask the nurse about preventing otitis media (OM). What should the nurse recommend? a. Avoid tobacco smoke. b. Use nasal decongestant. c. Avoid children with OM. d. Bottle-feed or breastfeed in supine position.

ANS: A Eliminating tobacco smoke from the child's environment is essential for preventing OM and other common childhood illnesses. Nasal decongestants are not useful in preventing OM. Children with uncomplicated OM are not contagious unless they show other upper respiratory infection symptoms. Children should be fed in an upright position to prevent OM.

An appropriate nursing intervention when caring for a child with pneumonia is to: a. Encourage rest. b. Encourage the child to lie on the unaffected side. c. Administer analgesics. d. Place the child in the Trendelenburg position.

ANS: A Encouraging rest by clustering care and promoting a quiet environment is the best intervention for a child with pneumonia. Lying on the affected side may promote comfort by splinting the chest and reducing pleural rubbing. Analgesics are not indicated. Children should be placed in a semi-erect position or position of comfort.

Which intervention should be included in the plan of care for an infant with the nursing diagnosis of Excess Fluid Volume related to congestive heart failure? a. Weigh the infant every day on the same scale at the same time. b. Notify the physician when weight gain exceeds more than 20 g/day. c. Put the infant in a car seat to minimize movement. d. Administer digoxin (Lanoxin) as ordered by the physician.

ANS: A Excess fluid volume may not be overtly visible. Weight changes may indicate fluid retention. Weighing the infant on the same scale at the same time each day ensures consistency. An excessive weight gain for an infant is an increase of more than 50 g/day. With fluid volume excess, skin will be edematous. The infant's position should be changed frequently to prevent undesirable pooling of fluid in certain areas. Lanoxin is used in the treatment of congestive heart failure to improve cardiac function. Diuretics will help the body get rid of excess fluid.

Which is the most commonly used method in completed suicides? a. Firearms c. Self-inflected laceration b. Drug overdose d. Carbon monoxide poisoning

ANS: A Firearms are the most commonly used instruments in completed suicides among both males and females. For adolescent boys, firearms are followed by hanging and overdose. For adolescent females, overdose and strangulation are the next most common means of completed suicide. The most common method of suicide attempt is overdose or ingestion of potentially toxic substances such as drugs. The second most common method of suicide attempt is self-inflicted laceration. Carbon monoxide poisoning is not one of the more frequent forms of suicide completion.

A young adolescent boy tells the nurse he "feels gawky." The nurse should explain that this occurs in adolescents because: a. Growth of the extremities and neck precedes growth in other areas. b. Growth is in the trunk and chest. c. The hip and chest breadth increases. d. The growth spurt occurs earlier in boys than it does in girls.

ANS: A Growth in length of the extremities and neck precedes growth in other areas, and, because these parts are the first to reach adult length, the hands and feet appear larger than normal during adolescence. Increases in hip and chest breadth take place in a few months, followed several months later by an increase in shoulder width. These changes are followed by increases in length of the trunk and depth of the chest. This sequence of changes is responsible for the characteristic long-legged, gawky appearance of early adolescent children. The growth spurt occurs earlier in girls than in boys.

Which statement is correct about childhood obesity? a. Heredity is an important factor in the development of obesity. b. Childhood obesity in the United States is decreasing. c. Childhood obesity is the result of inactivity. d. Childhood obesity can be attributed to an underlying disease in most cases.

ANS: A Heredity is an important fact that contributes to obesity. Identical twins reared apart tend to resemble their biologic parents to a greater extent than their adoptive parents. It is difficult to distinguish between hereditary and environmental factors. The rate of childhood obesity has increased so dramatically that it has now reached epidemic proportions. Inactivity is an important contributing factor; however, obesity is the result of a combination of a number of other factors. Fewer than 5% of all cases of obesity can be linked to underlying disease.

The nurse is administering an intravenous chemotherapeutic agent to a child with leukemia. The child suddenly begins to wheeze and have severe urticaria. Which is the most appropriate nursing action? a. Stop drug infusion immediately. b. Recheck rate of drug infusion. c. Observe child closely for next 10 minutes. d. Explain to child that this is an expected side effect.

ANS: A If an allergic reaction is suspected, the drug should be immediately discontinued. Any drug in the line should be withdrawn, and a normal saline infusion begun to keep the line open. Rechecking the rate of drug infusion, observing the child closely for next 10 minutes, and explaining to the child that this is an expected side effect can all be done after the drug infusion is stopped and the child is evaluated.

What is the priority nursing intervention for a child hospitalized with hemarthrosis resulting from hemophilia? a. Immobilization and elevation of the affected joint b. Administration of acetaminophen for pain relief c. Assessment of the child's response to hospitalization d. Assessment of the impact of hospitalization on the family system

ANS: A Immobilization and elevation of the joint will prevent further injury until bleeding is resolved. Although acetaminophen may help with pain associated with the treatment of hemarthrosis, it is not the priority nursing intervention. Assessment of a child's response to hospitalization is relevant to all hospitalized children; however, in this situation, psychosocial concerns are secondary to physiologic concerns. A priority nursing concern for this child is the management of hemarthrosis. Assessing the impact of hospitalization on the family system is relevant to all hospitalized children; however, it is not the priority in this situation.

Matt, age 14 years, seems to be always eating, although his weight is appropriate for his height. The best explanation for this is: a. This is normal because of increase in body mass. b. This is abnormal and suggestive of future obesity. c. His caloric intake would have to be excessive. d. He is substituting food for unfilled needs.

ANS: A In adolescence, nutritional needs are closely related to the increase in body mass. The peak requirements occur in the years of maximal growth. The caloric and protein requirements are higher than at almost any other time of life. This describes the expected eating pattern for young adolescents as long as weight and height are appropriate; obesity and substitution of food for unfilled needs are not concerns.

A nurse is conducting an in-service on asthma. Which statement is the most descriptive of bronchial asthma? a. There is heightened airway reactivity. b. There is decreased resistance in the airway. c. The single cause of asthma is an allergic hypersensitivity. d. It is inherited.

ANS: A In bronchial asthma, spasm of the smooth muscle of the bronchi and bronchioles causes constriction, producing impaired respiratory function. In bronchial asthma, there is increased resistance in the airway. There are multiple causes of asthma, including allergens, irritants, exercise, cold air, infections, medications, medical conditions, and endocrine factors. Atopy or development of an immunoglobulin E (IgE)-mediated response is inherited but is not the only cause of asthma.

In terms of cognitive development, the 5-year-old child would be expected to: a. Use magical thinking. b. Think abstractly. c. Understand conservation of matter. d. Be able to comprehend another person's perspective.

ANS: A Magical thinking is believing that thoughts can cause events. Abstract thought does not develop until school-age years. The concept of conservation is the cognitive task of school-age children ages 5 to 7 years. Five-year-olds cannot understand another's perspective.

Which statement characterizes moral development in older school-age children? a. They are able to judge an act by the intentions that prompted it rather than just by the consequences. b. Rules and judgments become more absolute and authoritarian. c. They view rule violations in an isolated context. d. They know the rules but cannot understand the reasons behind them.

ANS: A Older school-age children are able to judge an act by the intentions that prompted the behavior rather than just by the consequences. Rules and judgments become less absolute and authoritarian. Rule violation is likely to be viewed in relation to the total context in which it appears. Both the situation and the morality of the rule itself influence reactions.

Which painful, tender, pea-sized nodules may appear on the pads of the fingers or toes in bacterial endocarditis? a. Osler's nodes c. Subcutaneous nodules b. Janeway lesions d. Aschoff's nodules

ANS: A Osler's nodes are red, painful, intradermal nodes found on pads of the phalanges in bacterial endocarditis. Janeway lesions are painless hemorrhagic areas on palms and soles in bacterial endocarditis. Subcutaneous nodules are nontender swellings located over bony prominences, commonly found in rheumatic fever. Aschoff's nodules are small nodules composed of cells and leukocytes found in the interstitial tissues of the heart in rheumatic myocarditis.

What is the most common mode of transmission of human immunodeficiency virus (HIV) in the pediatric population? a. Perinatal transmission c. Blood transfusions b. Sexual abuse d. Poor hand washing

ANS: A Perinatal transmission accounts for the highest percentage (91%) of HIV infections in children. Infected women can transmit the virus to their infants across the placenta during pregnancy, at delivery, and through breastfeeding. Cases of HIV infection from sexual abuse have been reported; however, perinatal transmission accounts for most pediatric HIV infections. In the past some children became infected with HIV through blood transfusions; however, improved laboratory screening has significantly reduced the probability of contracting HIV from blood products. Poor hand washing is not an etiology of HIV infection.

A major clinical manifestation of rheumatic fever is: a. Polyarthritis. b. Osler's nodes. c. Janeway spots. d. Splinter hemorrhages of distal third of nails.

ANS: A Polyarthritis is swollen, hot, red, and painful joints. The affected joints will change every 1 to 2 days. Primarily the large joints are affected. Osler's nodes, Janeway spots, and splinter hemorrhages are characteristic of infective endocarditis.

A child with pulmonary atresia exhibits cyanosis with feeding. On reviewing this child's laboratory values, the nurse is not surprised to notice which abnormality? a. Polycythemia c. Dehydration b. Infection d. Anemia

ANS: A Polycythemia is a compensatory response to chronic hypoxia. The body attempts to improve tissue oxygenation by producing additional red blood cells and thereby increases the oxygen-carrying capacity of the blood. Infection is not a clinical consequence of cyanosis. Although dehydration can occur in cyanotic heart disease, it is not a compensatory mechanism for chronic hypoxia. It is not a clinical consequence of cyanosis. Anemia may develop as a result of increased blood viscosity. Anemia is not a clinical consequence of cyanosis.

An adolescent girl tells the nurse that she has suicidal thoughts. The nurse asks her if she has a specific plan. Asking this should be considered: a. An appropriate part of the assessment. b. Not a critical part of the assessment. c. Suggesting that the adolescent needs a plan. d. Encouraging the adolescent to devise a plan.

ANS: A Routine health assessments of adolescents should include questions that assess the presence of suicidal ideation or intent. Questions such as "Have you ever developed a plan to hurt yourself or kill yourself?" should be part of that assessment. Threats of suicide should always be taken seriously and evaluated. Suggesting that the adolescent needs a plan and encouraging her to devise this plan would be inappropriate statements by the nurse.

An inherited immunodeficiency disorder characterized by absence of both humoral and cell-mediated immunity is: a. Severe combined immunodeficiency syndrome (SCIDS). b. Acquired immunodeficiency syndrome. c. Wiskott-Aldrich syndrome. d. Fanconi syndrome.

ANS: A Severe SCIDS is a genetic disorder that results in deficits of both humoral and cellular immunity. Acquired immunodeficiency syndrome is not inherited. Wiskott-Aldrich syndrome is an X-linked recessive disorder with selected deficiencies of T and B lymphocytes. Fanconi syndrome is a hereditary disorder of red cell production.

A common characteristic of those who sexually abuse children is that they: a. Pressure the victim into secrecy. b. Are usually unemployed and unmarried. c. Are unknown to victims and victims' families. d. Have many victims that are each abused only once.

ANS: A Sex offenders may pressure the victim into secrecy, regarding the activity as a "secret between us" that other people may take away if they find out. Abusers are often employed upstanding members of the community. Most sexual abuse is committed by men and persons who are well known to the child. Abuse is often repeated with the same child over time. The relationship may start insidiously without the child realizing that sexual activity is part of the offer.

A 9-year-old girl often comes to the school nurse complaining of stomach pains. Her teacher says that she is completing her schoolwork satisfactorily, but lately she has been somewhat aggressive and stubborn in the classroom. The school nurse should recognize this as: a. Signs of stress. b. Developmental delay. c. A physical problem causing emotional stress. d. Lack of adjustment to the school environment.

ANS: A Signs of stress include stomach pains or headache, sleep problems, bed-wetting, changes in eating habits, aggressive or stubborn behavior, reluctance to participate, or regression to early behaviors. This child is exhibiting signs of stress, not developmental delay, a physical problem, or lack of adjustment.

Which is the most significant factor in distinguishing those who commit suicide from those who make suicidal attempts or threats? a. Social isolation c. Degree of depression b. Level of stress d. Desire to punish others

ANS: A Social isolation is a significant factor in distinguishing adolescents who will kill themselves from those who will not. It is also more characteristic of those who complete suicide than of those who make attempts or threats. Level of stress, degree of depression, and desire to punish others are contributing factors in suicide, but they are not the most significant factor in distinguishing those who complete suicide from those who attempt suicide.c

Teasing can be common during the school-age years. Which of the following does the nurse recognize as applying most to teasing? a. Can have a lasting effect on children b. Is not a significant threat to self-concept c. Is rarely based on anything that is concrete d. Is usually ignored by the child who is being teased

ANS: A Teasing in this age group is common and can have a long-lasting effect. Increasing awareness of differences, especially when accompanied by unkind comments and taunts from others, may make a child feel inferior and undesirable. Physical impairments such as hearing or visual defects, ears that "stick out," or birth marks assume great importance.

Which structural defects constitute tetralogy of Fallot? a. Pulmonic stenosis, ventricular septal defect, overriding aorta, right ventricular hypertrophy b. Aortic stenosis, ventricular septal defect, overriding aorta, right ventricular hypertrophy c. Aortic stenosis, atrial septal defect, overriding aorta, left ventricular hypertrophy d. Pulmonic stenosis, ventricular septal defect, aortic hypertrophy, left ventricular hypertrophy

ANS: A Tetralogy of Fallot has these four characteristics: pulmonary stenosis, ventricular septal defect, overriding aorta, and right ventricular hypertrophy. There is pulmonic stenosis but not aortic stenosis in tetralogy of Fallot. Right ventricular hypertrophy, not left ventricular hypertrophy, is present in tetralogy of Fallot. There is a ventricular septal defect, not an atrial septal defect, and overriding aorta, not aortic hypertrophy, is present.

Abdominal thrusts (the Heimlich maneuver) are recommended for airway obstruction in children older than: a. 1 year. c. 8 years. b. 4 years. d. 12 years.

ANS: A The Heimlich maneuver is recommended for airway obstruction in children older than 1 year. In children younger than 1 year, back blows and chest thrusts are administered.

The most appropriate nursing diagnosis for a child with anemia is: a. Activity Intolerance related to generalized weakness. b. Decreased Cardiac Output related to abnormal hemoglobin. c. Risk for Injury related to depressed sensorium. d. Risk for Injury related to dehydration and abnormal hemoglobin.

ANS: A The basic pathology in anemia is the decreased oxygen-carrying capacity of the blood. The nurse must assess the child's activity level (response to the physiologic state). The nursing diagnosis would reflect the activity intolerance. In generalized anemia no abnormal hemoglobin may be present. Only at a level of very severe anemia does cardiac output become altered. No decreased sensorium exists until profound anemia occurs. Dehydration and abnormal hemoglobin are not usually part of anemia.

Parents of a 3-year-old child with congenital heart disease are afraid to let their child play with other children because of possible overexertion. The nurse's reply should be based on knowing that: a. The child needs opportunities to play with peers. b. The child needs to understand that peers' activities are too strenuous. c. Parents can meet all the child's needs. d. Constant parental supervision is needed to avoid overexertion.

ANS: A The child needs opportunities for social development. Children usually limit their activities if allowed to set their own pace and regulate their activities. The child will limit activities as necessary. Parents must be encouraged to seek appropriate social activities for the child, especially before kindergarten. The child needs to have activities that foster independence.

The earliest recognizable clinical manifestation of cystic fibrosis (CF) is: a. Meconium ileus. b. History of poor intestinal absorption. c. Foul-smelling, frothy, greasy stools. d. Recurrent pneumonia and lung infections.

ANS: A The earliest clinical manifestation of CF is a meconium ileus, which is found in about 10% of children with CF. Clinical manifestations include abdominal distention, vomiting, failure to pass stools, and rapid development of dehydration. History of malabsorption is a later sign that manifests as failure to thrive. Foul-smelling stools and recurrent respiratory infections are later manifestations of CF.

A nurse is teaching adolescent boys about pubertal changes. The first sign of pubertal change seen with boys is: a. Testicular enlargement. c. Scrotal enlargement. b. Facial hair. d. Voice deepens.

ANS: A The first sign of pubertal changes in boys is testicular enlargement in response to testosterone secretion, which usually occurs in Tanner stage 2. Slight pubic hair is present and the smooth skin texture of the scrotum is somewhat altered. As testosterone secretion increases, the penis, testes, and scrotum enlarge. During Tanner stages 4 and 5, rising levels of testosterone cause the voice to deepen and facial hair appears at the corners of the upper lip and chin.

The nurse is admitting a child with rheumatic fever. Which therapeutic management should the nurse expect to implement? a. Administering penicillin b. Avoiding salicylates (aspirin) c. Imposing strict bed rest for 4 to 6 weeks d. Administering corticosteroids if chorea develops

ANS: A The goal of medical management is the eradication of the hemolytic streptococci. Penicillin is the drug of choice. Salicylates can be used to control the inflammatory process, especially in the joints, and reduce the fever and discomfort. Bed rest is recommended for the acute febrile stage, but it does not need to be strict. The chorea is transient and will resolve without treatment.

In which situation is there the greatest risk that a newborn infant will have a congenital heart defect (CHD)? a. Trisomy 21 detected on amniocentesis b. Family history of myocardial infarction c. Father has type 1 diabetes mellitus d. Older sibling born with Turner's syndrome

ANS: A The incidence of congenital heart disease is approximately 50% in children with trisomy 21 (Down syndrome). A family history of congenital heart disease, not acquired heart disease, increases the risk of giving birth to a child with CHD. Infants born to mothers who are insulin dependent have an increased risk of CHD. Infants identified as having certain genetic defects, such as Turner's syndrome, have a higher incidence of CHD.

The nurse is caring for an infant with congestive heart disease (CHD). The nurse should plan which intervention to decrease cardiac demands? a. Organize nursing activities to allow for uninterrupted sleep. b. Allow the infant to sleep through feedings during the night. c. Wait for the infant to cry to show definite signs of hunger. d. Discourage parents from rocking the infant

ANS: A The infant requires rest and conservation of energy for feeding. Every effort is made to organize nursing activities to allow for uninterrupted periods of sleep. Whenever possible, parents are encouraged to stay with their infant to provide the holding, rocking, and cuddling that help children sleep more soundly. To minimize disturbing the infant, changing bed linens and complete bathing are done only when necessary. Feeding is planned to accommodate the infant's sleep and wake patterns. The child is fed at the first sign of hunger, such as when sucking on fists, rather than waiting until he or she cries for a bottle because the stress of crying exhausts the limited energy supply. Because infants with CHD tire easily and may sleep through feedings, smaller feedings every 3 hours may be helpful.

A 14-year-old boy and his parents are concerned about bilateral breast enlargement. The nurse's discussion of this should be based on knowing that: a. This is usually benign and temporary. b. This is usually caused by Klinefelter's syndrome. c. Administration of estrogen effectively reduces gynecomastia. d. Administration of testosterone effectively reduces gynecomastia.

ANS: A The male breast responds to hormone changes. Some degree of bilateral or unilateral breast enlargement occurs frequently in boys during puberty. This is not a manifestation of Klinefelter's syndrome. Administration of estrogen or testosterone will have no effect on the reduction of breast tissue and may aggravate the condition.

The nurse is talking to a parent of an infant with heart failure about feeding the infant. Which statement about feeding the child is correct? a. "You may need to increase the caloric density of your infant's formula." b. "You should feed your baby every 2 hours." c. "You may need to increase the amount of formula your infant eats with each feeding." d. "You should place a nasal oxygen cannula on your infant during and after each feeding."

ANS: A The metabolic rate of infants with heart failure is greater because of poor cardiac function and increased heart and respiratory rates. Their caloric needs are greater than those of the average infants, yet their ability to take in the calories is diminished by their fatigue. Infants with heart failure should be fed every 3 hours; a 2-hour schedule does not allow for enough rest, and a 4-hour schedule is too long. Fluids must be carefully monitored because of the heart failure. Infants do not require supplemental oxygen with feedings.

A useful skill that the nurse should expect a 5-year-old child to be able to master is to: a. Tie shoelaces. b. Use a knife to cut meat. c. Hammer a nail. d. Make change from a quarter.

ANS: A Tying shoelaces is a fine motor task typical of 5-year-olds. Using a knife to cut meat is a fine motor task of a 7-year-old. Hammering a nail and making change from a quarter are fine motor tasks of an 8- to 9-year-old.

The school nurse is informed that a child with human immunodeficiency virus (HIV) will be attending school soon. Which is an important nursing intervention? a. Carefully follow universal precautions. b. Determine how the child became infected. c. Inform the parents of the other children. d. Reassure other children that they will not become infected.

ANS: A Universal precautions are necessary to prevent further transmission of the disease. It is not the role of the nurse to determine how the child became infected. Informing the parents of other children and reassuring other children that they will not become infected is a violation of the child's right to privacy.

Decongestant nose drops are recommended for a 10-month-old infant with an upper respiratory tract infection. Instructions for nose drops should include: a. Avoiding use for more than 3 days. b. Keeping drops to use again for nasal congestion. c. Administering drops until nasal congestion subsides. d. Administering drops after feedings and at bedtime

ANS: A Vasoconstrictive nose drops such as Neo-Synephrine should not be used for more than 3 days to avoid rebound congestion. Drops should be discarded after one illness because they may become contaminated with bacteria. Vasoconstrictive nose drops can have a rebound effect after 3 days of use. Drops administered before feedings are more helpful.

The nurse is caring for an adolescent brought to the hospital with acute drug toxicity. Cocaine is believed to be the drug involved. Data collection should include the: a. Mode of administration. b. Actual content of the drug. c. Function the drug plays in the adolescent's life. d. Adolescent's level of interest in rehabilitation.

ANS: A When the drug is questionable or unknown, every effort must be made to determine the type, amount of drug taken, the mode and time of administration, and factors relating to the onset of presenting symptoms. Because the actual content of most street drugs is highly questionable, this information would be difficult to obtain. It is helpful to know the pattern of use but not essential during this emergency. This is an inappropriate time for an evaluation about the level of interest in rehabilitation.

Which information should the nurse teach families about reducing exposure to pollens and dust (Select all that apply)? a. Replace wall-to-wall carpeting with wood and tile floors. b. Use an air conditioner. c. Put dust-proof covers on pillows and mattresses. d. Keep humidity in the house above 60%. e. Keep pets outside.

ANS: A, B, C Carpets retain dust. To reduce exposure to dust, carpeting should be replaced with wood, tile, slate, or vinyl. These floors can be cleaned easily. For anyone with pollen allergies, it is best to keep the windows closed and to run the air conditioner. Covering mattresses and pillows with dust-proof covers will reduce exposure to dust. A humidity level above 60% promotes dust mites. It is recommended that household humidity be kept between 40% and 50% to reduce dust mites inside the house. Keeping pets outside will help to decrease exposure to dander, but will not affect exposure to pollen and dust.

Peer victimization is becoming a significant problem for school-age children and adolescents in the United States. Parents should be educated regarding signs that a child is being bullied. These might include (select all that apply): a. The child spends an inordinate amount of time in the nurse's office. b. Belongings frequently go missing or are damaged. c. The child wants to be driven to school. d. School performance improves. e. The child freely talks about his or her day.

ANS: A, B, C Signs that may indicate a child is being bullied are similar to signs of other types of stress and include nonspecific illness or complaints, withdrawal, depression, school refusal, and decreased school performance. Children expressed fear of going to school or riding the school bus, and their belongings often are damaged or missing. Very often, children will not talk about what is happening to them.

Which demonstrates the school-age child's developing logic in the stage of concrete operations (select all that apply)? a. The school-age child is able to recognize that he can be a son, brother, or nephew at the same time. b. The school-age child understands the principles of adding, subtracting, and reversibility. c. The school-age child understands the principles of adding, subtracting, and reversibility. d. The school-age child has thinking that is characterized by egocentrism and animism.

ANS: A, B, C The school-age child understands that the properties of objects do not change when their order, form, or appearance does. Conservation occurs in the concrete operations stage. Comprehension of class inclusion occurs as the school-age child's logic increases. The child begins to understand that a person can be in more than one class at the same time. This is characteristic of concrete thinking and logical reasoning. The school-age child is able to understand principles of adding, subtracting, and the process of reversibility, which occurs in the stage of concrete operations. Thinking that is characterized by egocentrism and animism occurs in the intuitive thought stage, not the concrete operations stage of development.

Strict isolation is required for a child who is hospitalized with (select all that apply): a. Mumps. b. Chickenpox. c. Exanthema subitum (roseola). d. Erythema infectiosum (fifth disease). e. Parvovirus B19.

ANS: A, B, C, D Childhood communicable diseases requiring strict transmission-based precautions (Contact, Airborne, and Droplet Precautions) include diphtheria, chickenpox, measles, mumps, tuberculosis, adenovirus, Haemophilus influenzae type B, mumps, pertussis, plague, streptococcal pharyngitis, and scarlet fever. Strict isolation is not required for parvovirus B19.

Which play patterns does a 3-year-old child typically display (select all that apply)? a. Imaginary play b. Parallel play c. Cooperative play d. Structured play e. Associative play

ANS: A, B, C, E Children between ages 3 and 5 years enjoy parallel and associative play. Children learn to share and cooperate as they play in small groups. Play is often imitative, dramatic, and creative. Imaginary friends are common around age 3 years. Structured play is typical of school-age children.

Which toys should a nurse provide to promote imaginative play for a 3-year-old hospitalized child (select all that apply)? a. Plastic telephone b. Hand puppets c. Jigsaw puzzle (100 pieces) d. Farm animals and equipment e. Jump rope

ANS: A, B, D To promote imaginative play for a 3-year-old child, the nurse should provide: dress-up clothes, dolls and dollhouses, housekeeping toys, play-store toys, telephones, farm animals and equipment, village sets, trains, trucks, cars, planes, hand puppets, and medical kits. A 100-piece jigsaw puzzle and a jump rope would be appropriate for a young, school-age child but not a 3-year-old child.

A nurse is planning care for a 7-year-old child hospitalized with osteomyelitis. Which activities should the nurse plan to bring from the playroom for the child (select all that apply)? a. Paper and some paints b. Board games c. Jack-in-the-box d. Stuffed animals e. Computer games

ANS: A, B, E School-age children become fascinated with complex board, card, or computer games that they can play alone, with a best friend, or with a group. They also enjoy sewing, cooking, carpentry, gardening, and creative activities such as painting. Jack-in-the-box and stuffed animals would be appropriate for a toddler or preschool child.

A child has a total cholesterol level of 180 mg/dL. What dietary recommendations should the nurse make to the child and the child's parents (Select all that apply)? a. Replace whole milk with 2% or 1% milk b. Increase servings of red meat c. Increase servings of fish d. Avoid excessive intake of fruit juices e. Limit servings of whole grain

ANS: A, C, D A low-fat diet includes using nonfat or low-fat dairy products, limiting red meat intake, and increasing intake of fish, vegetables, whole grains, and legumes. Children should avoid excessive intake of fruit juices and other sweetened drinks, sugars, and saturated fats.

The nurse is preparing to give oral care to a school-age child with mucositis secondary to chemotherapy administered to treat leukemia. Which preparations should the nurse use for oral care on this child (Select all that apply)? a. Chlorhexidine gluconate (Peridex) b. Lemon glycerin swabs c. Antifungal troches (lozenges) d. Lip balm (Aquaphor) e. Hydrogen peroxide

ANS: A, C, D Preparations that may be used to prevent or treat mucositis include chlorhexidine gluconate (Peridex) because of its dual effectiveness against candidal and bacterial infections, antifungal troches (lozenges) or mouthwash, and lip balm (e.g., Aquaphor) to keep the lips moist. Agents that should not be used include lemon glycerin swabs (irritate eroded tissue and can decay teeth), hydrogen peroxide (delays healing by breaking down protein), and milk of magnesia (dries mucosa).

Which home care instructions should the nurse provide to the parents of a child with acquired immunodeficiency syndrome (AIDS) (Select all that apply)? a. Give supplemental vitamins as prescribed. b. Yearly influenza vaccination should be avoided. c. Administer trimethoprim-sulfamethoxazole (Bactrim) as prescribed. d. Notify the physician if the child develops a cough or congestion. e. Missed doses of antiretroviral medication do not need to be recorded.

ANS: A, C, D The parents should be taught that supplemental vitamins will be prescribed to aid in nutritional status. Bactrim is administered to prevent the opportunistic infection of Pneumocystis jiroveci pneumonia. The physician should be notified if the child with AIDS develops a cough and congestion. The yearly influenza vaccination is recommended, and any missed doses of antiretroviral medication need to be recorded and reported.

Research has shown that the most successful smoking cessation programs among teens include (select all that apply): a. Peer-led education and support. b. Information on the long-term effects of smoking. c. Programs including the media. d. School-based programs. e. Information on the immediate effects of smoking.

ANS: A, C, D, E Two areas of antismoking campaigns that have shown success are those that are peer-led and use media in education related to smoking prevention. School-based programs have also shown success and can be strengthened by expansion into the community and youth groups. Teens respond much better to education that focuses on the immediate effects of smoking. For the most part, smoking prevention programs that focus on the negative long-term effects of smoking have been ineffective.

A nurse is recommending strategies to a group of school-age children for prevention of obesity. Which should the nurse include (select all that apply)? a. Eat breakfast daily. b. Limit fruits and vegetables. c. Have frequent family meals with parents present. d. Eat frequently at restaurants. e. Limit television viewing to 2 hours a day.

ANS: A, C, E The nurse should counsel school-age children to eat breakfast daily, have mealtimes with family, and limit television viewing to 2 hours a day to prevent obesity. Fruits and vegetables should be consumed in the recommended quantities, and eating at restaurants should be limited.

An infant has developed staphylococcal pneumonia. Nursing care of the child with pneumonia includes which of the following? (Select all that apply). a. Cluster care to conserve energy b. Round-the-clock administration of antitussive agents c. Strict intake and output to avoid congestive heart failure d. Administration of antibiotics e. Placement in a mist tent

ANS: A, D Antibiotics are indicated for a bacterial pneumonia. Often the child will have decreased pulmonary reserve, and the clustering of care is essential. Round-the-clock antitussive agents and strict intake and output are not included in the care of the child with pneumonia. Mist tents are no longer utilized for pediatric respiratory care.

Injuries claim many lives during adolescence. Which factors contribute to early adolescents engaging in risk-taking behaviors (select all that apply)? a. Peer pressure b. A desire to master their environment c. Engagement in the process of separation from their parents d. A belief that they are invulnerable e. Impulsivity

ANS: A, D, E Peer pressure (including impressing peers) is a factor contributing to adolescent injuries. During early to middle adolescence, children feel that they are exempt from the consequences of risk-taking behaviors; they believe that negative consequences only happen to others. Feelings of invulnerability ("It can't happen to me") are evident in adolescence. Impulsivity places adolescents in unsafe situations. Mastering the environment is the task of young school-age children. Emancipation is a major issue for the older adolescent. The process is accomplished as the teenager gains an education or vocational training.

The nurse is caring for a hospitalized 4-year-old boy, Ryan. His parents tell the nurse that they will be back to visit at 6 PM. When Ryan asks the nurse when his parents are coming, the nurse's best response is: a. "They will be here soon." b. "They will come after dinner." c. "Let me show you on the clock when 6 PM is." d. "I will tell you every time I see you how much longer it will be."

ANS: B A 4-year-old understands time in relation to events such as meals. Children perceive "soon" as a very short time. The nurse may lose the child's trust if his parents do not return in the time he perceives as "soon." Children cannot read or use a clock for practical purposes until age 7 years. This answer assumes that the child understands the concept of hours and minutes, which is not developed until age 5 or 6 years.

Chelation therapy is begun on a child with b-thalassemia major. The purpose of this therapy is to: a. Treat the disease. c. Decrease the risk of hypoxia. b. Eliminate excess iron. d. Manage nausea and vomiting.

ANS: B A complication of the frequent blood transfusions in thalassemia is iron overload. Chelation therapy with deferoxamine (an iron-chelating agent) is given with oral supplements of vitamin C to increase iron excretion. Chelation therapy treats the side effects of disease management. Decreasing the risk of hypoxia and managing nausea and vomiting are not the purposes of chelation therapy.

The parent of a 4-year-old son tells the nurse that the child believes "monsters and the boogeyman" are in his bedroom at night. The nurse's best suggestion for coping with this problem is to: a. Insist that the child sleep with his parents until the fearful phase passes. b. Suggest involving the child to find a practical solution such as a night-light. c. Help the child understand that these fears are illogical. d. Tell the child frequently that monsters and the boogeyman do not exist.

ANS: B A night-light shows a child that imaginary creatures do not lurk in the darkness. Letting the child sleep with parents or telling the child that these creatures do not exist will not get rid of the fears. A 4-year-old is in the preconceptual age and cannot understand logical thought.

A school nurse is conducting a class with adolescents on suicide. Which true statement about suicide should the nurse include in the teaching session? a. A sense of hopelessness and despair are a normal part of adolescence. b. Gay and lesbian adolescents are at a particularly high risk for suicide. c. Problem-solving skills are of limited value to the suicidal adolescent. d. Previous suicide attempts are not an indication of risk for completed suicides.

ANS: B A significant number of teenage suicides occur among homosexual youths. Gay and lesbian adolescents who live in families or communities that do not accept homosexuality are likely to suffer low self-esteem, self-loathing, depression, and hopelessness as a result of a lack of acceptance from their family or community. At-risk teenagers include those who are depressed, have poor problem-solving skills, or use drugs and alcohol. History of previous suicide attempt is a serious indicator for possible suicide completion in the future.

Acyclovir (Zovirax) is given to children with chickenpox to: a. Minimize scarring. b. Decrease the number of lesions. c. Prevent aplastic anemia. d. Prevent spread of the disease.

ANS: B Acyclovir decreases the number of lesions, shortens duration of fever, and decreases itching, lethargy, and anorexia; however, it does not prevent scarring. Preventing aplastic anemia is not a function of acyclovir. Only quarantine of the infected child can prevent the spread of disease.

A young boy will receive a bone marrow transplant (BMT). This is possible because one of his older siblings is a histocompatible donor. This type of BMT is termed: a. Syngeneic. c. Monoclonal. b. Allogeneic. d. Autologous.

ANS: B Allogeneic transplants are from another individual. Because he and his sibling are histocompatible, the bone marrow transplantation can be done. Syngeneic marrow is from an identical twin. There is no such thing as a monoclonal bone marrow transplant. Autologous refers to the individual's own marrow.

Which statement is the most appropriate advice to give parents of a 16-year-old girl who is rebellious? a. "You need to be stricter so that your teen stops trying to test the limits." b. "You need to collaborate with your daughter and set limits that are perceived as being reasonable." c. "Increasing your teen's involvement with her peers will improve her self-esteem." d. "Allow your teenager to choose the type of discipline that is used in your home."

ANS: B Allowing teenagers to choose between realistic options and offering consistent and structured discipline typically enhances cooperation and decreases rebelliousness. Structure helps adolescents to feel more secure and assists them in the decision-making process. Setting stricter limits typically does not decrease rebelliousness or decrease testing of parental limits. Increasing peer involvement does not typically increase self-esteem.

A young child with human immunodeficiency virus is receiving several antiretroviral drugs. The purpose of these drugs is to: a. Cure the disease. b. Delay disease progression. c. Prevent spread of disease. d. Treat Pneumocystis jiroveci pneumonia.

ANS: B Although not a cure, these antiviral drugs can suppress viral replication, preventing further deterioration of the immune system, and delay disease progression. At this time cure is not possible. These drugs do not prevent the spread of the disease. Pneumocystis jiroveci prophylaxis is accomplished with antibiotics.

The nurse is caring for a child with carbon monoxide (CO) poisoning associated with smoke inhalation. What is essential in this child's care? a. Monitor pulse oximetry. b. Monitor arterial blood gases. c. Administer oxygen if respiratory distress develops. d. Administer oxygen if child's lips become bright, cherry red.

ANS: B Arterial blood gases and COHb levels are the best way to monitor CO poisoning. PaO2 monitored with pulse oximetry may be normal in the case of CO poisoning. Oxygen at 100% should be given as quickly as possible, not only if respiratory distress or other symptoms develop.

One of the most frequent causes of hypovolemic shock in children is: a. Myocardial infarction. c. Anaphylaxis. b. Blood loss. d. Congenital heart disease.

ANS: B Blood loss and extracellular fluid loss are two of the most frequent causes of hypovolemic shock in children. Myocardial infarction is rare in a child; if it occurred, the resulting shock would be cardiogenic, not hypovolemic. Anaphylaxis results in distributive shock from extreme allergy or hypersensitivity to a foreign substance. Congenital heart disease tends to contribute to hypervolemia, not hypovolemia.

Which statement is most descriptive of central nervous system stimulants? a. They produce strong physical dependence. b. They can result in strong psychologic dependence. c. Withdrawal symptoms are life threatening. d. Acute intoxication can lead to coma.

ANS: B Central nervous system stimulants such as amphetamines and cocaine produce a strong psychologic dependence. This class of drugs does not produce strong physical dependence and can be withdrawn without much danger. Acute intoxication leads to violent, aggressive behavior or psychotic episodes characterized by paranoia, uncontrollable agitation, and restlessness.

In terms of language and cognitive development, a 4-year-old child would be expected to: a. Think in abstract terms. b. Follow simple commands. c. Understand conservation of matter. d. Comprehend another person's perspective.

ANS: B Children ages 3 to 4 years can give and follow simple commands. Children cannot think abstractly at age 4 years. Conservation of matter is a developmental task of the school-age child. A 4-year-old child cannot comprehend another's perspective.

Which statement best describes why children have fewer respiratory tract infections as they grow older? a. The amount of lymphoid tissue decreases. b. Repeated exposure to organisms causes increased immunity. c. Viral organisms are less prevalent in the population. d. Secondary infections rarely occur after viral illnesses.

ANS: B Children have increased immunity after exposure to a virus. The amount of lymphoid tissue increases as children grow older. Viral organisms are not less prevalent, but older children have the ability to resist invading organisms. Secondary infections after viral illnesses include Mycoplasma pneumoniae and groups A and B streptococcal infections.

The psychologic effects of being obese during adolescence include: a. Sexual promiscuity. b. Poor body image. c. Memory having no effect on eating behavior. d. Accurate body image but self-deprecating attitude.

ANS: B Common emotional consequences of obesity include poor body image, low self-esteem, social isolation, and feelings of depression and isolation. Sexual promiscuity is an unlikely effect of obesity. The obese adolescent often substitutes food for affection. Eating behaviors are closely related to memory. Memory and appetite are closely linked and can be modified over time with treatment. Obese adolescents most often have a very poor self-image.

As part of the treatment for congestive heart failure, the child takes the diuretic furosemide. As part of teaching home care, the nurse encourages the family to give the child foods such as bananas, oranges, and leafy vegetables. These foods are recommended because they are high in: a. Chlorides. c. Sodium. b. Potassium. d. Vitamins.

ANS: B Diuretics that work on the proximal and distal renal tubules contribute to increased losses of potassium. The child's diet should be supplemented with potassium.

Which consideration is the most important in managing tuberculosis (TB) in children? a. Skin testing annually c. Adequate nutrition b. Pharmacotherapy d. Adequate hydration

ANS: B Drug therapy for TB includes isoniazid, rifampin, and pyrazinamide daily for 2 months and 2 or 3 times a week for the remaining 4 months. Although skin testing and adequate nutrition and hydration are important, pharmacotherapy is the most important intervention for TB.

Which behavior suggests appropriate psychosocial development in the adolescent? a. The adolescent seeks validation for socially acceptable behavior from older adults. b. The adolescent is self-absorbed and self-centered and has sudden mood swings. c. Adolescents move from peers and enjoy spending time with family members. d. Conformity with the peer group increases in late adolescence.

ANS: B During adolescence, energy is focused within. Adolescents concentrate on themselves in an effort to determine who they are or who they will be. Adolescents are likely to be impulsive and impatient. Parents often describe their teenager as being "self-centered" or "lazy." The peer group validates acceptable behavior during adolescence. Adolescents move from family and enjoy spending time with peers. Adolescents also spend time alone; they need this time to think and concentrate on themselves. Conformity becomes less important in late adolescence.

Which predisposes the adolescent to feel an increased need for sleep? a. An inadequate diet b. Rapid physical growth c. Decreased activity that contributes to a feeling of fatigue d. The lack of ambition typical of this age group

ANS: B During growth spurts, the need for sleep is increased. Rapid physical growth, the tendency toward overexertion, and the overall increased activity of this age contribute to fatigue.

Which statement best describes fear in school-age children? a. They are increasingly fearful for body safety. b. Most of the new fears that trouble them are related to school and family. c. They should be encouraged to hide their fears to prevent ridicule by peers. d. Those who have numerous fears need continuous protective behavior by parents to eliminate these fears.

ANS: B During the school-age years, children experience a wide variety of fears, but new fears related predominantly to school and family bother children during this time. During the middle-school years, children become less fearful of body safety than they were as preschoolers. Parents and other persons involved with children should discuss their fear with them individually or as a group activity. Sometimes school-age children hide their fears to avoid being teased. Hiding the fears does not end them and may lead to phobias.

Which type of croup is always considered a medical emergency? a. Laryngitis c. Spasmodic croup b. Epiglottitis d. Laryngotracheobronchitis (LTB)

ANS: B Epiglottitis is always a medical emergency needing antibiotics and airway support for treatment. Laryngitis is a common viral illness in older children and adolescents, with hoarseness and upper respiratory infection symptoms. Spasmodic croup is treated with humidity. LTB may progress to a medical emergency in some children.

An adolescent being seen by the nurse practitioner for a sports physical is identified as having hypertension. On further testing, it is discovered the child has a cardiac abnormality. The initial treatment of secondary hypertension initially involves: a. Weight control and diet. b. Treating the underlying disease. c. Administration of digoxin. d. Administration of b-adrenergic receptor blockers.

ANS: B Identification of the underlying disease should be the first step in treating secondary hypertension. Weight control and diet are nonpharmacologic treatments for primary hypertension. Digoxin is indicated in the treatment of congestive heart failure. b-Adrenergic receptor blockers are indicated in the treatment of primary hypertension.

The nurse is assessing a child with acute epiglottitis. Examining the child's throat by using a tongue depressor might precipitate which symptom or condition? a. Inspiratory stridor c. Sore throat b. Complete obstruction d. Respiratory tract infection

ANS: B If a child has acute epiglottitis, examination of the throat may cause complete obstruction and should be performed only when immediate intubation can take place. Stridor is aggravated when a child with epiglottitis is supine. Sore throat and pain on swallowing are early signs of epiglottitis. Epiglottitis is caused by Haemophilus influenzae in the respiratory tract.

What is descriptive of the preschooler's understanding of time? a. Has no understanding of time b. Associates time with events c. Can tell time on a clock d. Uses terms like "yesterday" appropriately

ANS: B In a preschooler's understanding, time has a relation with events such as, "We'll go outside after lunch." Preschoolers develop an abstract sense of time at age 3 years. Children can tell time on a clock at age 7 years. Children do not fully understand use of time-oriented words until age 6 years.

Cardiopulmonary resuscitation is begun on a toddler. Which pulse is usually palpated because it is the most central and accessible? a. Radial c. Femoral b. Carotid d. Brachial

ANS: B In a toddler, the carotid pulse is palpated. The radial pulse is not considered a central pulse. The femoral pulse is not the most central and accessible. The brachial pulse is felt in infants younger than 1 year.

The nurse is caring for a child with persistent hypoxia secondary to a cardiac defect. The nurse recognizes that a risk of cerebrovascular accidents (strokes) exists. An important objective to decrease this risk is to: a. Minimize seizures. c. Promote cardiac output. b. Prevent dehydration. d. Reduce energy expenditure.

ANS: B In children with persistent hypoxia, polycythemia develops. Dehydration must be prevented in hypoxemic children because it potentiates the risk of strokes. Minimizing seizures, promoting cardiac output, and reducing energy expenditure will not reduce the risk of cerebrovascular accidents.

Which statement accurately describes physical development during the school-age years? a. The child's weight almost triples. b. A child grows an average of 2 inches per year. c. Few physical differences are apparent among children at the end of middle childhood. d. Fat gradually increases, which contributes to the child's heavier appearance.

ANS: B In middle childhood, growth in height and weight occur at a slower pace. Between the ages of 6 and 12 years, children grow 2 inches per year. In middle childhood, children's weight will almost double; they gain 3 kg/year. At the end of middle childhood, girls grow taller and gain more weight than boys. Children take on a slimmer look with longer legs in middle childhood.

Which clinical changes occur as a result of septic shock? a. Hypothermia c. Vasoconstriction b. Increased cardiac output d. Angioneurotic edema

ANS: B Increased cardiac output, which results in warm, flushed skin, is one of the manifestations of septic shock. Fever and chills are characteristic of septic shock. Vasodilation is more common in septic shock. Angioneurotic edema occurs as a manifestation in anaphylactic shock.

The nurse is performing an assessment on a child and notes the presence of Koplik's spots. In which communicable disease are Koplik's spots present? a. Rubella b. Measles (rubeola) c. Chickenpox (varicella) d. Exanthema subitum (roseola)

ANS: B Koplik's spots are small, irregular red spots with a minute, bluish white center found on the buccal mucosa 2 days before systemic rash. Koplik's spots are not present with rubella, varicella, or roseola.

The nurse is caring for a child with acute respiratory distress syndrome (ARDS) associated with sepsis. Nursing actions should include: a. Force fluids. c. Institute seizure precautions. b. Monitor pulse oximetry. d. Encourage a high-protein diet.

ANS: B Monitoring cardiopulmonary status is an important evaluation tool in the care of the child with ARDS. Maintenance of vascular volume and hydration is important and should be done parenterally. Seizures are not a side effect of ARDS. Adequate nutrition is necessary, but a high-protein diet is not helpful.

A teen asks a nurse, "What is physical dependence in substance abuse?" Which is the correct response by the nurse? a. Problem that occurs in conjunction with addiction b. Involuntary physiologic response to drug c. Culturally defined use of drugs for purposes other than accepted medical purposes d. Voluntary behavior based on psychosocial needs

ANS: B Physical dependence is an involuntary response to the pharmacologic characteristics of drugs such as opioids or alcohol. A person can be physically dependent on a narcotic/drug without being addicted; for example, patients who use opioids to control pain need increasing doses to achieve the same effect. Dependence is a physiologic response; it is not culturally determined or subject to voluntary control.

What is descriptive of the play of school-age children? a. Individuality in play is better tolerated than at earlier ages. b. Knowing the rules of a game gives an important sense of belonging. c. They like to invent games, making up the rules as they go. d. Team play helps children learn the universal importance of competition and winning.

ANS: B Play involves increased physical skill, intellectual ability, and fantasy. Children form groups and cliques and develop a sense of belonging to a team or club. At this age, children begin to see the need for rules. Conformity and ritual permeate their play. Their games have fixed and unvarying rules, which may be bizarre and extraordinarily rigid. With team play, children learn about competition and the importance of winning, an attribute highly valued in the United States.

A 4-year-old boy is hospitalized with a serious bacterial infection. He tells the nurse that he is sick because he was "bad." The nurse's best interpretation of this comment is that it is: a. A sign of stress. b. Common at this age. c. Suggestive of maladaptation. d. Suggestive of excessive discipline at home.

ANS: B Preschoolers cannot understand the cause and effect of illness. Their egocentrism makes them think that they are directly responsible for events, making them feel guilt for things outside of their control. Children of this age show stress by regressing developmentally or acting out. Maladaptation is unlikely. This comment does not imply excessive discipline at home.

Parents of a 12-year-old child ask the clinic nurse, "How many hours of sleep should our child get?' The nurse should respond that 12-year-old children need how many hours of sleep at night? a. 8 b. 9 c. 10 d. 11

ANS: B School-age children usually do not require naps, but they do need to sleep approximately 11 hours at age 5 years and 9 hours at age 12 years each night.

A condition in which the normal adult hemoglobin is partly or completely replaced by abnormal hemoglobin is: a. Aplastic anemia. c. Thalassemia major. b. Sickle cell anemia. d. Iron deficiency anemia.

ANS: B Sickle cell anemia is one of a group of diseases collectively called hemoglobinopathies, in which normal adult hemoglobin is replaced by abnormal hemoglobin. Aplastic anemia is a lack of cellular elements being produced. Hemophilia refers to a group of bleeding disorders in which there is deficiency of one of the factors necessary for coagulation. Iron deficiency anemia affects size and depth of color of hemoglobin and does not involve abnormal hemoglobin.

The nurse is completing a health history with a 16-year-old male. He informs the nurse that he has started using smokeless tobacco after he plays baseball. Which information regarding smokeless tobacco would be most correct for the nurse to provide to this teen? a. Not addicting. b. Proven to be carcinogenic. c. Easy to stop using. d. A safe alternative to cigarette smoking.

ANS: B Smokeless tobacco is a popular substitute for cigarettes and poses serious health hazards to children and adolescents. Smokeless tobacco is associated with cancer of the mouth and jaw. Smokeless tobacco is just as addictive as cigarettes. Although teens believe that it is easy to stop using smokeless tobacco, this is not the case. A popular belief is that smokeless tobacco is a safe alternative to cigarettes; this has been proven incorrect. Half of all teens who use smokeless tobacco agree that it poses significant health risks.

The parent of a toddler calls the nurse, asking about croup. What is a distinguishing manifestation of spasmodic croup? a. Wheezing is heard audibly. c. It is bacterial in nature. b. It has a harsh, barky cough. d. The child has a high fever.

ANS: B Spasmodic croup is viral in origin, is usually preceded by several days of symptoms of upper respiratory tract infection, and often begins at night. It is marked by a harsh, metallic, barky cough; sore throat; inspiratory stridor; and hoarseness. Wheezing is not a distinguishing manifestation of croup. It can accompany conditions such as asthma or bronchiolitis. A high fever is not usually present.

A normal characteristic of the language development of a preschool-age child is: a. Lisp. b. Stammering. c. Echolalia. d. Repetition without meaning.

ANS: B Stammering and stuttering are normal dysfluencies in preschool-age children. Lisps are not a normal characteristic of language development. Echolalia and repetition are traits of toddlers' language.

A nurse is teaching nursing students the physiology of congenital heart defects. Which defect results in decreased pulmonary blood flow? a. Atrial septal defect c. Ventricular septal defect b. Tetralogy of Fallot d. Patent ductus arteriosus

ANS: B Tetralogy of Fallot results in decreased blood flow to the lungs. The pulmonic stenosis increases the pressure in the right ventricle, causing the blood to go from right to left across the interventricular septal defect. Atrial and ventricular septal defects and patent ductus arteriosus result in increased pulmonary blood flow.

A nurse is assessing an older school-age child recently admitted to the hospital. Which assessment indicates that the child is in an appropriate stage of cognitive development? a. The child's addition and subtraction ability b. The child's ability to classify c. The child's vocabulary d. The child's play activity

ANS: B The ability to classify things from simple to complex and the ability to identify differences and similarities are cognitive skills of the older school-age child; this demonstrates use of classification and logical thought processes. Subtraction and addition are appropriate cognitive activities for the young school-age child. Vocabulary is not as valid an assessment of cognitive ability as is the child's ability to classify. Play activity is not as valid an assessment of cognitive function as is the ability to classify.

A 17-year-old tells the nurse that he is not having sex because it would make his parents very angry. This response indicates that the adolescent has a developmental lag in which area? a. Cognitive development c. Psychosocial development b. Moral development d. Psychosexual development

ANS: B The appropriate moral development for a 17-year-old would include evidence that the teenager has internalized a value system and does not depend on parents to determine right and wrong behaviors. Adolescents who remain concrete thinkers may never advance beyond conformity to please others and avoid punishment. Cognitive development is related to moral development, but it is not the pivotal point in determining right and wrong behaviors. Identity formation is the psychosocial development task. Energy is focused within the adolescent, who exhibits behavior that is self-absorbed and egocentric. Although a task during adolescence is the development of a sexual identity, the teenager's dependence on the parents' sanctioning of right or wrong behavior is more appropriately related to moral development.

The mean age of menarche in the United States is: a. 11.5 years c. 13.5 years b. 12.5 years d. 14 years

ANS: B The average age of menarche is 12 years and 4 months in North American girls, with a normal range of 10.5 to 15 years.

When preparing a school-age child and the family for heart surgery, the nurse should consider: a. Not showing unfamiliar equipment. b. Letting child hear the sounds of an electrocardiograph monitor. c. Avoiding mentioning postoperative discomfort and interventions. d. Explaining that an endotracheal tube will not be needed if the surgery goes well.

ANS: B The child and family should be exposed to the sights and sounds of the intensive care unit. All positive, nonfrightening aspects of the environment are emphasized. The child should be shown unfamiliar equipment, and its use should be demonstrated on a doll. Carefully prepare the child for the postoperative experience, including intravenous lines, incision, and endotracheal tube.

The nurse is preparing an adolescent for discharge after a cardiac catheterization. Which statement by the adolescent would indicate a need for further teaching? a. "I should avoid tub baths but may shower." b. "I have to stay on strict bed rest for 3 days." c. "I should remove the pressure dressing the day after the procedure." d. "I may attend school but should avoid exercise for several days."

ANS: B The child does not need to be on strict bed rest for 3 days. Showers are recommended; children should avoid a tub bath. The pressure dressing is removed the day after the catheterization and replaced by an adhesive bandage to keep the area clean. Strenuous activity must be avoided for several days, but the child can return to school.

Which immunization should be given with caution to children infected with human immunodeficiency virus? a. Influenza c. Pneumococcus b. Varicella d. Inactivated poliovirus

ANS: B The children should be carefully evaluated before giving live viral vaccines such as varicella, measles, mumps, and rubella. The child must be immunocompetent and not have contact with other severely immunocompromised individuals. Influenza, pneumococcus, and inactivated poliovirus are not live vaccines.

The nurse assessing a premature newborn infant auscultates a continuous machinery-like murmur. This finding is associated with which congenital heart defect? a. Pulmonary stenosis c. Ventricular septal defect b. Patent ductus arteriosus d. Coarctation of the aorta

ANS: B The classic murmur associated with patent ductus arteriosus is a machinery-like one that can be heard throughout both systole and diastole. A systolic ejection murmur that may be accompanied by a palpable thrill is a manifestation of pulmonary stenosis. The characteristic murmur associated with ventricular septal defect is a loud, harsh, holosystolic murmur. A systolic murmur that is accompanied by an ejection click may be heard on auscultation when coarctation of the aorta is present.

The nurse encourages the mother of a toddler with acute laryngotracheobronchitis to stay at the bedside as much as possible. The nurse's rationale for this action is primarily that: a. Mothers of hospitalized toddlers often experience guilt. b. The mother's presence will reduce anxiety and ease the child's respiratory efforts. c. Separation from the mother is a major developmental threat at this age. d. The mother can provide constant observations of the child's respiratory efforts.

ANS: B The family's presence will decrease the child's distress. The mother may experience guilt, but this is not the best answer. Although separation from the mother is a developmental threat for toddlers, the main reason to keep parents at the child's bedside is to ease anxiety and therefore respiratory effort. The child should have constant cardiorespiratory monitoring and noninvasive oxygen saturation monitoring, but the parent should not play this role in the hospital.

An 8-month-old infant has a hypercyanotic spell while blood is being drawn. The nurse's first action should be to: a. Assess for neurologic defects. b. Place the child in the knee-chest position. c. Begin cardiopulmonary resuscitation. d. Prepare the family for imminent death.

ANS: B The first action is to place the infant in the knee-chest position. Blow-by oxygen may be indicated. Neurologic defects are unlikely. The child should be assessed for airway, breathing, and circulation. Often calming the child and administering oxygen and morphine can alleviate the hypercyanotic spell; cardiopulmonary resuscitation is not necessary, and death is unlikely.

A boy with leukemia screams whenever he needs to be turned or moved. The most probable cause of this pain is: a. Edema. c. Petechial hemorrhages. b. Bone involvement. d. Changes within the muscles.

ANS: B The invasion of the bone marrow with leukemic cells gradually causes a weakening of the bone and a tendency toward fractures. As leukemic cells invade the periosteum, increasing pressure causes severe pain. Edema, petechial hemorrhages, and muscular changes would not cause severe pain.

A school-age child is admitted in vaso-occlusive sickle cell crisis. The child's care should include: a. Correction of acidosis. b. Adequate hydration and pain management. c. Pain management and administration of heparin. d. Adequate oxygenation and replacement of factor VIII.

ANS: B The management of crises includes adequate hydration, minimizing energy expenditures, pain management, electrolyte replacement, and blood component therapy if indicated. The acidosis will be corrected as the crisis is treated. Heparin and factor VIII are not indicated in the treatment of vaso-occlusive sickle cell crisis. Oxygen may prevent further sickling, but it is not effective in reversing sickling because it cannot reach the clogged blood vessels.

An important consideration for the school nurse who is planning a class on bicycle safety is: a. Most bicycle injuries involve collision with an automobile. b. Head injuries are the major causes of bicycle-related fatalities. c. Children should wear bicycle helmets if they ride on paved streets. d. Children should not ride double unless the bicycle has an extra-large seat.

ANS: B The most important aspect of bicycle safety is to encourage the rider to use a protective helmet. Head injuries are the major cause of bicycle-related fatalities. Although motor vehicle collisions do cause injuries to bicyclists, most injuries result from falls. The child should always wear a properly fitted helmet approved by the U.S. Consumer Product Safety Commission. Children should not ride double.

The primary nursing intervention necessary to prevent bacterial endocarditis is to: a. Institute measures to prevent dental procedures. b. Counsel parents of high risk children about prophylactic antibiotics. c. Observe children for complications such as embolism and heart failure. d. Encourage restricted mobility in susceptible children.

ANS: B The objective of nursing care is to counsel the parents of high risk children about both the need for prophylactic antibiotics for dental procedures and the necessity of maintaining excellent oral health. The child's dentist should be aware of the child's cardiac condition. Dental procedures should be done to maintain a high level of oral health. Prophylactic antibiotics are necessary. Observing for complications and encouraging restricted mobility in susceptible children should be done, but maintaining good oral health and using prophylactic antibiotics are most important.

The parents of a child hospitalized with sickle cell anemia tell the nurse that they are concerned about narcotic analgesics causing addiction. The nurse should explain that narcotic analgesics: a. Are often ordered but not usually needed. b. Rarely cause addiction because they are medically indicated. c. Are given as a last resort because of the threat of addiction. d. Are used only if other measures such as ice packs are ineffective.

ANS: B The pain of sickle cell anemia is best treated by a multidisciplinary approach. Mild-to-moderate pain can be controlled by ibuprofen and acetaminophen. When narcotics are indicated, they are titrated to effect and given around the clock. Patient-controlled analgesia reinforces the patient's role and responsibility in managing the pain and provides flexibility in dealing with pain. Few if any patients who receive opioids for severe pain become behaviorally addicted to the drug. Narcotics are often used because of the severe nature of the pain of vaso-occlusive crisis. Ice is contraindicated because of its vasoconstrictive effects.

The leading cause of death after heart transplantation is: a. Infection. c. Cardiomyopathy. b. Rejection. d. Congestive heart failure.

ANS: B The posttransplantation course is complex. The leading cause of death after cardiac transplantation is rejection. Infection is a continued risk secondary to the immunosuppression necessary to prevent rejection. Cardiomyopathy is one of the indications for cardiac transplant. Congestive heart failure is not a leading cause of death.

Which behavior by parents or teachers will best assist the child in negotiating the developmental task of industry? a. Identifying failures immediately and asking the child's peers for feedback b. Structuring the environment so the child can master tasks c. Completing homework for children who are having difficulty in completing assignments d. Decreasing expectations to eliminate potential failures

ANS: B The task of the caring teacher or parent is to identify areas in which a child is competent and to build on successful experiences to foster feelings of mastery and success. Structuring the environment to enhance self-confidence and to provide the opportunity to solve increasingly more complex problems will promote a sense of mastery. Asking peers for feedback reinforces the child's feelings of failure. When teachers or parents complete children's homework for them, it sends the message that they do not trust the children to do a good job. Providing assistance and suggestions and praising their best efforts are more appropriate. Decreasing expectations to eliminate failures will not promote a sense of achievement or mastery.

What is the appropriate priority nursing action for the infant with a CHD who has an increased respiratory rate, is sweating, and is not feeding well? a. Recheck the infant's blood pressure. b. Alert the physician. c. Withhold oral feeding. d. Increase the oxygen rate.

ANS: B These are signs of early congestive heart failure, and the physician should be notified. Although rechecking blood pressure may be indicated, it is not the priority action. Withholding the infant's feeding is an incomplete response to the problem. Increasing oxygen may alleviate symptoms; however, medications such as digoxin and furosemide are necessary to improve heart function and fluid retention. Notifying the physician is the priority nursing action.

b-Adrenergic agonists and methylxanthines are often prescribed for a child with an asthma attack. What is their action? a. Liquefy secretions c. Reduce inflammation of the lungs b. Dilate the bronchioles d. Reduce infection

ANS: B These medications work to dilate the bronchioles in acute exacerbations. These medications do not liquefy secretions or reduce infection. Corticosteroids and mast cell stabilizers reduce inflammation in the lungs.

Parents tell the nurse that they found their 3-year-old daughter and a male cousin of the same age inspecting each other closely as they used the bathroom. Which is the most appropriate recommendation the nurse should make? a. Punish children so this behavior stops. b. Neither condone nor condemn the curiosity. c. Allow children unrestricted permission to satisfy this curiosity. d. Get counseling for this unusual and dangerous behavior.

ANS: B Three-year-olds become aware of anatomic differences and are concerned about how the other "works." Such exploration should not be condoned or condemned. Children should not be punished for this normal exploration. Encouraging the children to ask questions of the parents and redirecting their activity are more appropriate than giving permission. Exploration is age-appropriate and not dangerous behavior.

Which statement, made by a 4-year-old child's father, is true about the care of the preschooler's teeth? a. "Because the 'baby teeth' are not permanent, they are not important to the child." b. "My son can be encouraged to brush his teeth after I have thoroughly cleaned his teeth." c. "My son's 'permanent teeth' will begin to come in at 4 to 5 years of age." d. "Fluoride supplements can be discontinued when my son's 'permanent teeth' erupt."

ANS: B Toddlers and preschoolers lack the manual dexterity to remove plaque adequately, so parents must assume this responsibility. Deciduous teeth are important because they maintain spacing and play an important role in the growth and development of the jaws and face and in speech development. Secondary teeth erupt at about 6 years of age. If the family does not live in an area where fluoride is included in the water supply, fluoride supplements should be continued.

According to Erikson, the psychosocial task of adolescence is developing: a. Intimacy. c. Initiative. b. Identity. d. Independence.

ANS: B Traditional psychosocial theory holds that the developmental crises of adolescence lead to the formation of a sense of identity. Intimacy is the developmental stage for early adulthood. Initiative is the developmental stage for early childhood. Independence is not one of Erikson's developmental stages.

A child with cystic fibrosis is receiving recombinant human deoxyribonuclease (rhDNase). This drug: a. May cause mucus to thicken. b. May cause voice alterations. c. Is given subcutaneously. d. Is not indicated for children younger than 12 years.

ANS: B Two of the only adverse effects of rhDNase are voice alterations and laryngitis. rhDNase decreases viscosity of mucus, is given in an aerosolized form, and is safe for children younger than 12 years of age.

Asthma in infants is usually triggered by: a. Medications. c. Exposure to cold air. b. A viral infection. d. Allergy to dust or dust mites.

ANS: B Viral illnesses cause inflammation that causes increased airway reactivity in asthma. Medications such as aspirin, nonsteroidal antiinflammatory drugs, and antibiotics may aggravate asthma, but not frequently in infants. Exposure to cold air may exacerbate already existing asthma. Allergy is associated with asthma, but 20% to 40% of children with asthma have no evidence of allergic disease.

The nurse is evaluating a child who is taking digoxin for her cardiac condition. The nurse is cognizant that a common sign of digoxin toxicity is: a. Seizures. c. Bradypnea. b. Vomiting. d. Tachycardia.

ANS: B Vomiting is a common sign of digoxin toxicity. Seizures are not associated with digoxin toxicity. The child will have a slower heart rate, not respiratory rate.

When teaching injury prevention during the school-age years, the nurse should include: a. Teaching the need to fear strangers. b. Teaching basic rules of water safety. c. Avoiding letting children cook in microwave ovens. d. Cautioning children against engaging in competitive sports.

ANS: B Water safety instruction is an important source of injury prevention at this age. The child should be taught to swim, select safe and supervised places to swim, swim with a companion, check sufficient water depth for diving, and use an approved flotation device. Teach stranger safety, not fearing strangers. This includes not going with strangers, not having personalized clothing in public places, having children tell parents if anyone makes them uncomfortable, and teaching children to say "no" in uncomfortable situations. Teach children safe cooking methods. Caution against engaging in hazardous sports, such as those involving trampolines.

When is a child with chickenpox considered to be no longer contagious? a. When fever is absent b. When lesions are crusted c. 24 hours after lesions erupt d. 8 days after onset of illness

ANS: B When the lesions are crusted, the chickenpox is no longer contagious. This may be a week after onset of disease. The child is still contagious once the fever has subsided and after the lesions erupt, and may or may not be contagious any time after 6 days as long as all lesions are crusted over.

The nurse is planning care for a school-age child admitted to the hospital with hemophilia. Which interventions should the nurse plan to implement for this child (Select all that apply)? a. Fingersticks for blood work instead of venipunctures b. Avoidance of intramuscular (IM) injections c. Acetaminophen (Tylenol) for mild pain control d. Soft toothbrush for dental hygiene e. Administration of packed red blood cells

ANS: B, C, D Nurses should take special precautions when caring for a child with hemophilia to prevent the use of procedures that may cause bleeding, such as IM injections. The subcutaneous route is substituted for IM injections whenever possible. Venipunctures for blood samples are usually preferred for these children. There is usually less bleeding after the venipuncture than after finger or heel punctures. Neither aspirin nor any aspirin-containing compound should be used. Acetaminophen is a suitable aspirin substitute, especially for controlling mild pain. A soft toothbrush is recommended for dental hygiene to prevent bleeding from the gums. Packed red blood cells are not administered. The primary therapy for hemophilia is replacement of the missing clotting factor. The products available are factor VIII concentrates.

A nurse teaches parents that team play is important for school-age children. Which can children develop by experiencing team play (select all that apply)? a. Achieve personal goals over group goals. b. Learn complex rules. c. Experience competition. d. Learn about division of labor.

ANS: B, C, D Team play helps stimulate cognitive growth because children are called on to learn many complex rules, make judgments about those rules, plan strategies, and assess the strengths and weaknesses of members of their own team and members of the opposing team. Team play can also contribute to children's social, intellectual, and skill growth. Children work hard to develop the skills needed to become team members, to improve their contribution to the group, and to anticipate the consequences of their behavior for the group. Team play teaches children to modify or exchange personal goals for goals of the group; it also teaches them that division of labor is an effective strategy for attaining a goal.

The nurse is caring for a child with aplastic anemia. Which nursing diagnoses are appropriate (Select all that apply)? a. Acute Pain related to vaso-occlusion b. Risk for Infection related to inadequate secondary defenses or immunosuppression c. Ineffective Protection related to thrombocytopenia d. Ineffective Tissue Perfusion related to anemia e. Ineffective Protection related to abnormal clotting

ANS: B, C, D These are appropriate nursing diagnosis for the nurse planning care for a child with aplastic anemia. Aplastic anemia is a condition in which the bone marrow ceases production of the cells it normally manufactures, resulting in pancytopenia. The child will have varying degrees of the disease depending on how low the values are for absolute neutrophil count (affecting the body's response to infection), platelet count (putting the child at risk for bleeding), and absolute reticulocyte count (causing the child to have anemia). Acute Pain related to vaso-occlusion is an appropriate nursing diagnosis for sickle cell anemia for the child in vaso-occlusive crisis, but it is not applicable to a child with aplastic anemia. Ineffective Protection related to abnormal clotting is an appropriate diagnosis for a child with hemophilia.

Which should the nurse teach about prevention of sickle cell crises to parents of a preschool child with sickle cell disease (Select all that apply)? a. Limit fluids at bedtime. b. Notify the health care provider if a fever of 38.5° C (101.3° F) or greater occurs. c. Give penicillin as prescribed. d. Use ice packs to decrease the discomfort of vaso-occlusive pain in the legs. e. Notify the health care provider if your child begins to develop symptoms of a cold.

ANS: B, C, E The most important issues to teach the family of a child with sickle cell anemia are to (1) seek early intervention for problems, such as a fever of 38.5° C (101.3° F) or greater; (2) give penicillin as ordered; (3) recognize signs and symptoms of splenic sequestration, as well as respiratory problems that can lead to hypoxia; and (4) treat the child normally. The nurse emphasizes the importance of adequate hydration to prevent sickling and to delay the adhesion-stasis-thrombosis-ischemia cycle. It is not sufficient to advise parents to "force fluids" or "encourage drinking." They need specific instructions on how many daily glasses or bottles of fluid are required. Many foods are also a source of fluid, particularly soups, flavored ice pops, ice cream, sherbet, gelatin, and puddings. Increased fluids combined with impaired kidney function result in the problem of enuresis. Parents who are unaware of this fact frequently use the usual measures to discourage bedwetting, such as limiting fluids at night. Enuresis is treated as a complication of the disease, such as joint pain or some other symptom, to alleviate parental pressure on the child. Ice should not be used during a vaso-occlusive pain crisis because it vasoconstricts and impairs circulation even more.

Which screening tests should the school nurse perform for the adolescent (select all that apply)? a. Glucose b. Vision c. Hearing d. Cholesterol e. Scoliosis

ANS: B, C, E The school nurse should perform vision, hearing, and scoliosis screening tests according to the school district's required schedule. Glucose and cholesterol screening would be performed in the medical clinic setting.

Parents of a school-age child with hemophilia ask the nurse, "Which sports are recommended for children with hemophilia?" Which sports should the nurse recommend (Select all that apply)? a. Soccer b. Swimming c. Basketball d. Golf e. Bowling

ANS: B, D, E Because almost all persons with hemophilia are boys, the physical limitations in regard to active sports may be a difficult adjustment, and activity restrictions must be tempered with sensitivity to the child's emotional and physical needs. Use of protective equipment, such as padding and helmets, is particularly important, and noncontact sports, especially swimming, walking, jogging, tennis, golf, fishing, and bowling, are encouraged. Contact sports such as soccer and basketball are not recommended.

In terms of language and cognitive development, a 4-year-old child would be expected to have which traits (select all that apply)? a. Think in abstract terms. b. Follow directional commands. c. Understand conservation of matter. d. Use sentences of eight words. e. Tell exaggerated stories.

ANS: B, E Children ages 3 to 4 years can give and follow simple commands and tell exaggerated stories. Children cannot think abstractly at age 4 years. Conservation of matter is a developmental task of the school-age child. Five-year-old children use sentences with eight words with all parts of speech.

Which condition is caused by a virus that primarily infects a specific subset of T lymphocytes, the CD4+ T-cells? a. Wiskott-Aldrich syndrome b. Idiopathic thrombocytopenic purpura (ITP) c. Acquired immunodeficiency syndrome (AIDS) d. Severe combined immunodeficiency disease

ANS: C AIDS is caused by the human immunodeficiency virus, which primarily attacks the CD4+ T-cells. Wiskott-Aldrich syndrome, ITP, and severe combined immunodeficiency disease are not viral illnesses.

A nurse is caring for an adolescent hospitalized for cellulitis. The nurse notes that the adolescent experiences many "mood swings" throughout the day. The nurse interprets this behavior as: a. Requiring a referral to a mental health counselor. b. Requiring some further lab testing. c. Normal behavior. d. Related to feelings of depression.

ANS: C Adolescents vacillate in their emotional states between considerable maturity and childlike behavior. One minute they are exuberant and enthusiastic; the next minute they are depressed and withdrawn. Because of these mood swings, adolescents are frequently labeled as unstable, inconsistent, and unpredictable, but the behavior is normal. The behavior would not require a referral to a mental health counselor or further lab testing. The mood swings do not indicate depression.

It is generally recommended that a child with acute streptococcal pharyngitis can return to school: a. When the sore throat is better. c. After taking antibiotics for 24 hours. b. If no complications develop. d. After taking antibiotics for 3 days.

ANS: C After children have taken antibiotics for 24 hours, even if the sore throat persists, they are no longer contagious to other children. Complications may take days to weeks to develop.

A child is brought to the emergency department experiencing an anaphylactic reaction to a bee sting. While an airway is being established, what medication should the nurse prepare for immediate administration? a. Diphenhydramine (Benadryl) c. Epinephrine b. Dopamine d. Calcium chloride

ANS: C After the first priority of establishing an airway, epinephrine is the drug of choice. Benadryl is not a strong enough antihistamine for this severe a reaction. Dopamine and calcium chloride are not appropriate drugs for this type of reaction.

A preschool child is scheduled for an echocardiogram. Parents ask the nurse whether they can hold the child during the procedure. The nurse should answer with which response? a. "You will be able to hold your child during the procedure." b. "Your child can be active during the procedure, but can't sit in your lap." c. "Your child must lie quietly; sometimes a mild sedative is administered before the procedure." d. "The procedure is invasive so your child will be restrained during the echocardiogram."

ANS: C Although an echocardiogram is noninvasive, painless, and associated with no known side effects, it can be stressful for children. The child must lie quietly in the standard echocardiographic positions; crying, nursing, being held, or sitting up often leads to diagnostic errors or omissions. Therefore, infants and young children may need a mild sedative; older children benefit from psychologic preparation for the test. The distraction of a video or movie is often helpful.

A child is diagnosed with influenza, probably type A disease. Management includes: a. Clear liquid diet for hydration. b. Aspirin to control fever. c. Amantadine hydrochloride to reduce symptoms. d. Antibiotics to prevent bacterial infection.

ANS: C Amantadine hydrochloride may reduce symptoms related to influenza type A if administered within 24 to 48 hours of onset. It is ineffective against type B or C. A clear liquid diet is not necessary for influenza, but maintaining hydration is important. Aspirin is not recommended in children because of increased risk of Reye's syndrome. Acetaminophen or ibuprofen is a better choice. Preventive antibiotics are not indicated for influenza unless there is evidence of a secondary bacterial infection.

Which child should the nurse document as being anemic? a. 7-year-old child with a hemoglobin of 11.5 g/dL b. 3-year-old child with a hemoglobin of 12 g/dL c. 14-year-old child with a hemoglobin of 10 g/dL d. 1-year-old child with a hemoglobin of 13 g/dL

ANS: C Anemia is a condition in which the number of red blood cells, or hemoglobin concentration, is reduced below the normal values for age. Anemia is defined as a hemoglobin level below 10 or 11 g/dL. The child with a hemoglobin of 10 g/dL would be considered anemic. The normal hemoglobin for a child after 2 years of age is 11.5 to 15.5 g/dL.

Which type of play is most typical of the preschool period? a. Solitary b. Parallel c. Associative d. Team

ANS: C Associative play is group play in similar or identical activities but without rigid organization or rules. Solitary play is that of infants. Parallel play is that of toddlers. School-age children play in teams.

Which defect results in increased pulmonary blood flow? a. Pulmonic stenosis c. Atrial septal defect b. Tricuspid atresia d. Transposition of the great arteries

ANS: C Atrial septal defect results in increased pulmonary blood flow. Blood flows from the left atrium (higher pressure) into the right atrium (lower pressure) and then to the lungs via the pulmonary artery. Pulmonic stenosis is an obstruction to blood flowing from the ventricles. Tricuspid atresia results in decreased pulmonary blood flow. Transposition of the great arteries results in mixed blood flow.

A nurse is reviewing hormone changes that occur during adolescence. The hormone that is responsible for the growth of beard, mustache, and body hair in the male is: a. Estrogen. c. Androgen. b. Pituitary hormone. d. Progesterone.

ANS: C Beard, mustache, and body hair on the chest, upward along the linea alba, and sometimes on other areas (e.g., back and shoulders) appears in males and is androgen dependent. Estrogen and progesterone are produced by the ovaries in the female and do not contribute to body hair appearance in the male. The pituitary hormone does not have any relationship to body hair appearance in the male.

The nurse is planning activity for a 4-year-old child with anemia. Which activity should the nurse plan for this child? a. Game of "hide and seek" in the children's outdoor play area b. Participation in dance activities in the playroom c. Puppet play in the child's room d. A walk down to the hospital lobby

ANS: C Because the basic pathologic process in anemia is a decrease in oxygen-carrying capacity, an important nursing responsibility is to assess the child's energy level and minimize excess demands. The child's level of tolerance for activities of daily living and play is assessed, and adjustments are made to allow as much self-care as possible without undue exertion. Puppet play in the child's room would not be overly tiring. Hide and seek, dancing, and walking to the lobby would not conserve the anemic child's energy.

An adolescent will receive a bone marrow transplant (BMT). The nurse should explain that the bone marrow will be administered by which route? a. Bone grafting c. Intravenous infusion b. Bone marrow injection d. Intraabdominal infusion

ANS: C Bone marrow from a donor is infused intravenously, not intraabdominally, and the transfused stem cells will repopulate the marrow. Because the stem cells migrate to the recipient's marrow when given intravenously, this method of administration is used rather than bone grafting or bone marrow injection.

What describes moral development in younger school-age children? a. The standards of behavior now come from within themselves. b. They do not yet experience a sense of guilt when they misbehave. c. They know the rules and behaviors expected of them but do not understand the reasons behind them. d. They no longer interpret accidents and misfortunes as punishment for misdeeds.

ANS: C Children who are ages 6 and 7 years know the rules and behaviors expected of them but do not understand the reasons for them. Young children do not believe that standards of behavior come from within themselves but that rules are established and set down by others. Younger school-age children learn standards for acceptable behavior, act according to these standards, and feel guilty when they violate them. Misfortunes and accidents are viewed as punishment for bad acts.

During the preschool period, the emphasis of injury prevention should be placed on: a. Constant vigilance and protection. b. Punishment for unsafe behaviors. c. Education for safety and potential hazards. d. Limitation of physical activities.

ANS: C Education for safety and potential hazards is appropriate for preschoolers because they can begin to understand dangers. Constant vigilance and protection is not practical at this age since preschoolers are becoming more independent. Punishment may make children scared of trying new things. Limitation of physical activities is not appropriate.

The parents of a 15-year-old girl are concerned that their adolescent spends too much time looking in the mirror. Which statement is the most appropriate for the nurse to make? a. "Your teenager needs clearer and stricter limits about her behavior." b. "Your teenager needs more responsibility at home." c. "During adolescence this behavior is not unusual." d. "The behavior is abnormal and needs further investigation."

ANS: C Egocentric and narcissistic behavior is normal during this period of development. The teenager is seeking a personal identity. Stricter limits are not an appropriate response for a behavior that is part of normal development. More responsibility at home is not an appropriate response for this situation. The behavior is normal and needs no further investigation.

Vitamin A supplementation may be recommended for the young child who has: a. Mumps. b. Rubella. c. Measles (rubeola). d. Erythema infectiosum.

ANS: C Evidence shows that vitamin A decreases morbidity and mortality associated with measles. Vitamin A will not lessen the effects of mumps, rubella, or fifth disease.

A nurse is teaching parents of first-grade children general guidelines to assist their children in adapting to school. Which statement by the parents indicates they understand the teaching? a. "We will only meet with the teacher if problems occur." b. "We will discourage hobbies so our child focuses on schoolwork." c. "We will plan a trip to the library as often as possible." d. "We will expect our child to make all As in school."

ANS: C General guidelines for parents to help their child in school include sharing an interest in reading. The library should be used frequently and books the child is reading should be discussed. Hobbies should be encouraged. The parents should not expect all As. They should focus on growth more than grades.

Children receiving long-term systemic corticosteroid therapy are most at risk for: a. Hypotension. b. Dilation of blood vessels in the cheeks. c. Growth delays. d. Decreased appetite and weight loss.

ANS: C Growth delay is associated with long-term steroid use. Hypertension is a clinical manifestation of long-term systemic steroid administration. Dilation of blood vessels in the cheeks is associated with an excess of topically administered steroids. Increased appetite and weight gain are clinical manifestations of excess systemic corticosteroid therapy.

The school nurse tells adolescents in the clinic that confidentiality and privacy will be maintained unless a life-threatening situation arises. This practice is: a. Not appropriate in a school setting. b. Never appropriate because adolescents are minors. c. Important in establishing trusting relationships. d. Suggestive that the nurse is meeting his or her own needs.

ANS: C Health professionals who work with adolescents should consider the adolescents' increasing independence and responsibility while maintaining privacy and ensuring confidentiality. However, circumstances may occur in which they are not able to maintain confidentiality, such as self-destructive behavior or maltreatment by others. Confidentiality and privacy are necessary to facilitate trust with this age group. The nurse must be aware of the limits placed on confidentiality by local jurisdiction.

The nurse is caring for a child after heart surgery. What should she or he do if evidence is found of cardiac tamponade? a. Increase analgesia. b. Apply warming blankets. c. Immediately report this to the physician. d. Encourage the child to cough, turn, and breathe deeply.

ANS: C If evidence is noted of cardiac tamponade (blood or fluid in the pericardial space constricting the heart), the physician is notified immediately of this life-threatening complication. Increasing analgesia may be done before the physician drains the fluid, but the physician must be notified. Warming blankets are not indicated at this time. Encouraging the child to cough, turn, and breathe deeply should be deferred until after the evaluation by the physician.

An important nursing consideration when suctioning a young child who has had heart surgery is to: a. Perform suctioning at least every hour. b. Suction for no longer than 30 seconds at a time. c. Administer supplemental oxygen before and after suctioning. d. Expect symptoms of respiratory distress when suctioning.

ANS: C If suctioning is indicated, supplemental oxygen is administered with a manual resuscitation bag before and after the procedure to prevent hypoxia. Suctioning should be done only as indicated, not on a routine basis. The child should be suctioned for no more than 5 seconds at one time. Symptoms of respiratory distress are to be avoided by using the appropriate technique.

Which statement best describes b-thalassemia major (Cooley's anemia)? a. All formed elements of the blood are depressed. b. Inadequate numbers of red blood cells are present. c. Increased incidence occurs in families of Mediterranean extraction. d. Increased incidence occurs in persons of West African descent.

ANS: C Individuals who live near the Mediterranean Sea and their descendants have the highest incidence of thalassemia. An overproduction of red cells occurs. Although numerous, the red cells are relatively unstable. Sickle cell disease is common in blacks of West African descent.

Young people with anorexia nervosa are often described as being: a. Independent. c. Conforming. b. Disruptive. d. Low achieving.

ANS: C Individuals with anorexia nervosa are described as perfectionist, academically high achievers, conforming, and conscientious. Independent, disruptive, and low achieving are not part of the behavioral characteristics of anorexia nervosa.

In providing anticipatory guidance to parents whose child will soon be entering kindergarten, which is a critical factor in preparing a child for kindergarten entry? a. The child's ability to sit still b. The child's sense of learned helplessness c. The parent's interactions and responsiveness to the child d. Attending a preschool program

ANS: C Interactions between the parent and child are an important factor in the development of academic competence. Parental encouragement and support maximize a child's potential. The child's ability to sit still is important to learning; however, parental responsiveness and involvement are more important factors. Learned helplessness is the result of a child feeling that he or she has no effect on the environment and that his or her actions do not matter. Parents who are actively involved in a supportive learning environment will demonstrate a more positive approach to learning. Preschool and day care programs can supplement the developmental opportunities provided by parents at home, but they are not critical in preparing a child for entering kindergarten.

Iron dextran is ordered for a young child with severe iron deficiency anemia. Nursing considerations include: a. Administering with meals. b. Administering between meals. c. Injecting deeply into a large muscle. d. Massaging injection site for 5 minutes after administration of drug.

ANS: C Iron dextran is a parenteral form of iron. When administered intramuscularly, it must be injected into a large muscle using the Z-track method. Iron dextran is for intramuscular or intravenous administration; it is not taken orally. The site should not be massaged to prevent leakage, potential irritation, and staining of the skin.

An important nursing consideration when chest tubes will be removed from a child is to: a. Explain that it is not painful. b. Explain that only a Band-Aid will be needed. c. Administer analgesics before the procedure. d. Expect bright red drainage for several hours after removal.

ANS: C It is appropriate to prepare the child for the removal of chest tubes with analgesics. Short-acting medications can be used that are administered through an existing intravenous line. It is not a pain-free procedure. A sharp, momentary pain is felt, and this should not be misrepresented to the child. A petroleum gauze/airtight dressing is needed. Little or no drainage should be found on removal.

What is most descriptive of the pathophysiology of leukemia? a. Increased blood viscosity occurs. b. Thrombocytopenia (excessive destruction of platelets) occurs. c. Unrestricted proliferation of immature white blood cells (WBCs) occurs. d. The first stage of the coagulation process is abnormally stimulated.

ANS: C Leukemia is a group of malignant disorders of the bone marrow and the lymphatic system. It is defined as an unrestricted proliferation of immature WBCs in the blood-forming tissues of the body. Increased blood viscosity may occur secondary to the increased number of WBCs. Thrombocytopenia may occur secondary to the overproduction of WBCs in the bone marrow. The coagulation process is unaffected by leukemia.

Which action by the school nurse is important in the prevention of rheumatic fever? a. Encourage routine cholesterol screenings. b. Conduct routine blood pressure screenings. c. Refer children with sore throats for throat cultures. d. Recommend salicylates instead of acetaminophen for minor discomforts.

ANS: C Nurses have a role in prevention—primarily in screening school-age children for sore throats caused by group A b-hemolytic streptococci. They can achieve this by actively participating in throat culture screening or by referring children with possible streptococcal sore throats for testing. Cholesterol and blood pressure screenings do not facilitate the recognition and treatment of group A b-hemolytic streptococci. Salicylates should be avoided routinely because of the risk of Reye's syndrome after viral illnesses.

A nurse planning care for a school-age child should take into account that which thought process is seen at this age? a. Animism b. Magical thinking c. Ability to conserve d. Thoughts are all-powerful

ANS: C One cognitive task of school-age children is mastering the concept of conservation. At an early age (5 to 7 years), children grasp the concept of reversibility of numbers as a basis for simple mathematics problems (e.g., 2 + 4 = 6 and 6 - 4 = 2). They learn that simply altering their arrangement in space does not change certain properties of the environment, and they are able to resist perceptual cues that suggest alterations in the physical state of an object. Animism, magical thinking, and believing that thoughts are all-powerful are thought processes seen in preschool children.

A group of boys ages 9 and 10 years have formed a "boys-only" club that is open to neighborhood and school friends who have skateboards. This should be interpreted as: a. Behavior that encourages bullying and sexism. b. Behavior that reinforces poor peer relationships. c. Characteristic of social development of this age. d. Characteristic of children who later are at risk for membership in gangs.

ANS: C One of the outstanding characteristics of middle childhood is the creation of formalized groups or clubs. Peer-group identification and association are essential to a child's socialization. Poor relationships with peers and a lack of group identification can contribute to bullying. A boys-only club does not have a direct correlation with later gang activity.

Imaginary playmates are beneficial to the preschool child because they: a. Take the place of social interactions. b. Take the place of pets and other toys. c. Become friends in times of loneliness. d. Accomplish what the child has already successfully accomplished.

ANS: C One purpose of an imaginary friend is to be a friend in time of loneliness. Imaginary friends do not take the place of social interactions but may encourage conversation. Imaginary friends do not take the place of pets or toys. They accomplish what the child is still attempting, not what has already been accomplished.

A clinical manifestation of the systemic venous congestion that can occur with congestive heart failure is: a. Tachypnea. c. Peripheral edema. b. Tachycardia. d. Pale, cool extremities.

ANS: C Peripheral edema, especially periorbital edema, is a clinical manifestation of systemic venous congestion. Tachypnea is a manifestation of pulmonary congestion. Tachycardia and pale, cool extremities are clinical manifestations of impaired myocardial function.

Several blood tests are ordered for a preschool child with severe anemia. She is crying and upset because she remembers the venipuncture done at the clinic 2 days ago. The nurse should explain that: a. Venipuncture discomfort is very brief. b. Only one venipuncture will be needed. c. Topical application of local anesthetic can eliminate venipuncture pain. d. Most blood tests on children require only a finger puncture because a small amount of blood is needed.

ANS: C Preschool children are very concerned about both pain and the loss of blood. When preparing the child for venipuncture, a topical anesthetic will be used to eliminate any pain. This is a very traumatic experience for preschool children. They are concerned about their bodily integrity. A local anesthetic should be used, and a bandage should be applied to maintain bodily integrity. A promise that only one venipuncture will be needed should not be made in case multiple attempts are required. Both finger punctures and venipunctures are traumatic for children. Both require preparation.

A 4-year-old child tells the nurse that she does not want another blood sample drawn because "I need all my insides, and I don't want anyone taking them out." Which is the nurse's best interpretation of this? a. Child is being overly dramatic. b. Child has a disturbed body image. c. Preschoolers have poorly defined body boundaries. d. Preschoolers normally have a good understanding of their bodies.

ANS: C Preschoolers have little understanding of body boundaries, which leads to fears of mutilation. The child is not capable of being dramatic at 4 years of age. She truly has fear. Body image is just developing in the school-age child. Preschoolers do not have good understanding of their bodies.

A nurse is teaching an adolescent about primary hypertension. The nurse knows that which of the following is correct? a. Primary hypertension should be treated with diuretics as soon as it is detected. b. Congenital heart defects are the most common cause of primary hypertension. c. Primary hypertension may be treated with weight reduction. d. Primary hypertension is not affected by exercise.

ANS: C Primary hypertension in children may be treated with weight reduction and exercise programs. If ineffective, pharmacologic intervention may be needed. Primary hypertension is considered an inherited disorder.

Generally the earliest age at which puberty begins is: a. 13 years in girls, 13 years in boys. b. 11 years in girls, 11 years in boys. c. 10 years in girls, 12 years in boys. d. 12 years in girls, 10 years in boys.

ANS: C Puberty signals the beginning of the development of secondary sex characteristics. This begins in girls earlier than in boys. Usually a 2-year difference occurs in the age at onset. Girls and boys do not usually begin puberty at the same age; girls usually begin earlier than boys do.

Which statement by the nurse is most appropriate to a 15-year-old whose friend has mentioned suicide? a. "Tell your friend to come to the clinic immediately." b. "You need to gather details about your friend's suicide plan." c. "Your friend's threat needs to be taken seriously and immediate help for your friend is important." d. "If your friend mentions suicide a second time, you will want to get your friend some help."

ANS: C Suicide is the third most common cause of death among American adolescents. A suicide threat from an adolescent serves as a dramatic message to others and should be taken seriously. Adolescents at risk should be targeted for supportive guidance and counseling before a crisis occurs. Instructing a 15-year-old to tell a friend to come to the clinic immediately provides the teen with limited information and does not address the concern. It is important to determine whether a person threatening suicide has a plan of action; however, the best information for the 15-year-old to have is that all threats of suicide should be taken seriously and immediate help is important. Taking time to gather details or waiting until the teen discusses it a second time may be too late.

A parent whose two school-age children have asthma asks the nurse in what sports, if any, they can participate. The nurse should recommend: a. Soccer. c. Swimming. b. Running. d. Basketball.

ANS: C Swimming is well tolerated in children with asthma because they are breathing air fully saturated with moisture and because of the type of breathing required in swimming. Exercise-induced bronchospasm is more common in sports that involve endurance, such as soccer, running, and basketball. Prophylaxis with medications may be necessary.

Which intervention for treating croup at home should be taught to parents? a. Have a decongestant available to give the child when an attack occurs. b. Have the child sleep in a dry room. c. Take the child outside. d. Give the child an antibiotic at bedtime.

ANS: C Taking the child into the cool, humid, night air may relieve mucosal swelling and improve symptoms. Decongestants are inappropriate for croup, which affects the middle airway level. A dry environment may contribute to symptoms. Croup is caused by a virus. Antibiotic treatment is not indicated.

The nurse is explaining Tanner staging to an adolescent and her mother. Which statement best describes Tanner staging? a. Predictable stages of puberty that are based on chronologic age b. Staging of puberty based on the initiation of menarche and nocturnal emissions c. Predictable stages of puberty that are based on primary and secondary sexual characteristics d. Staging of puberty based on the initiation of primary sexual characteristics

ANS: C Tanner sexual-maturing ratings are based on the development of stages of primary and secondary sexual characteristics. Tanner stages are not based on chronologic age. The age at which an adolescent enters puberty is variable. The puberty stage in girls begins with breast development. The puberty stage in boys begins with genital enlargement. Primary sexual characteristics are not the sole basis of Tanner staging.

Identify the statement that is the most accurate about moral development in the 9-year-old school-age child. a. Right and wrong are based on physical consequences of behavior. b. The child obeys parents because of fear of punishment. c. The school-age child conforms to rules to please others. d. Parents are the determiners of right and wrong for the school-age child.

ANS: C The 7- to 12-year-old child bases right and wrong on a good-boy or good-girl orientation in which the child conforms to rules to please others and avoid disapproval. Children 4 to 7 years of age base right and wrong on consequences, the most important consideration for this age-group. Parents determine right and wrong for the child younger than 4 years of age.

Which statement most accurately describes the pathologic changes of sickle cell anemia? a. Sickle-shaped cells carry excess oxygen. b. Sickle-shaped cells decrease blood viscosity. c. Increased red blood cell destruction occurs. d. Decreased red blood cell destruction occurs.

ANS: C The clinical features of sickle cell anemia are primarily the result of increased red blood cell destruction and obstruction caused by the sickle-shaped red blood cells. Sickled red cells have decreased oxygen-carrying capacity and transform into the sickle shape in conditions of low oxygen tension. When the sickle cells change shape, they increase the viscosity in the area where they are involved in the microcirculation.

What is best described as the inability of the heart to pump an adequate amount of blood to the systemic circulation at normal filling pressures? a. Pulmonary congestion c. Congestive heart failure b. Congenital heart defect d. Systemic venous congestion

ANS: C The definition of congestive heart failure is the inability of the heart to pump an adequate amount of blood to the systemic circulation at normal filling pressures to meet the metabolic demands of the body. Pulmonary congestion is an excessive accumulation of fluid in the lungs. Congenital heart defect is a malformation of the heart present at birth. Systemic venous congestion is an excessive accumulation of fluid in the systemic vasculature.

It is now recommended that children with asthma who are taking long-term inhaled steroids should be assessed frequently because they may develop: a. Cough. c. Slowed growth. b. Osteoporosis. d. Cushing's syndrome

ANS: C The growth of children on long-term inhaled steroids should be assessed frequently to assess for systemic effects of these drugs. Cough is prevented by inhaled steroids. No evidence exists that inhaled steroids cause osteoporosis. Cushing's syndrome is caused by long-term systemic steroids.

As related to inherited disorders, which statement is descriptive of most cases of hemophilia? a. Autosomal dominant disorder causing deficiency in a factor involved in the blood-clotting reaction b. X-linked recessive inherited disorder causing deficiency of platelets and prolonged bleeding c. X-linked recessive inherited disorder in which a blood-clotting factor is deficient d. Y-linked recessive inherited disorder in which the red blood cells become moon shaped

ANS: C The inheritance pattern in 80% of all of the cases of hemophilia is X-linked recessive. The two most common forms of the disorder are factor VIII deficiency (hemophilia A or classic hemophilia), and factor IX deficiency (hemophilia B or Christmas disease). The disorder involves coagulation factors, not platelets. The disorder does not involve red cells or the Y chromosome.

A school-age child with leukemia experienced severe nausea and vomiting when receiving chemotherapy for the first time. The most appropriate nursing action to prevent or minimize these reactions with subsequent treatments is to: a. Encourage drinking large amounts of favorite fluids. b. Encourage child to take nothing by mouth (remain NPO) until nausea and vomiting subside. c. Administer an antiemetic before chemotherapy begins. d. Administer an antiemetic as soon as child has nausea.

ANS: C The most beneficial regimen to minimize nausea and vomiting associated with chemotherapy is to administer the antiemetic before the chemotherapy is begun. The goal is to prevent anticipatory symptoms. Drinking fluids will add to the discomfort of the nausea and vomiting. Encouraging the child to remain NPO will help with this episode, but the child will have the discomfort and be at risk for dehydration. Administering an antiemetic after the child has nausea does not avoid anticipatory nausea.

The parents of a child with cancer tell the nurse that a bone marrow transplant (BMT) may be necessary. What should the nurse recognize as important when discussing this with the family? a. BMT should be done at time of diagnosis. b. Parents and siblings of child have a 25% chance of being a suitable donor. c. Finding a suitable donor involves matching antigens from the human leukocyte antigen (HLA) system. d. If BMT fails, chemotherapy or radiotherapy must be continued.

ANS: C The most successful BMTs come from suitable HLA-matched donors. The timing of a BMT depends on the disease process involved. It usually follows intensive high-dose chemotherapy and/or radiation therapy. Usually parents only share approximately 50% of the genetic material with their children. A one-in-four chance exists that two siblings will have two identical haplotypes and will be identically matched at the HLA loci. Discussing the continuation of chemotherapy or radiotherapy is not appropriate when planning the BMT. That decision will be made later.

When teaching the mother of a 9-month-old infant about administering liquid iron preparations, the nurse should include that: a. They should be given with meals. b. They should be stopped immediately if nausea and vomiting occur. c. Adequate dosage will turn the stools a tarry green color. d. Preparation should be allowed to mix with saliva and bathe the teeth before swallowing.

ANS: C The nurse should prepare the mother for the anticipated change in the child's stools. If the iron dose is adequate, the stools will become a tarry green color. The lack of the color change may indicate insufficient iron. The iron should be given in two divided doses between meals, when the presence of free hydrochloric acid is greatest. Iron is absorbed best in an acidic environment. Vomiting and diarrhea may occur with iron administration. If these occur, the iron should be given with meals, and the dosage reduced and gradually increased as the child develops tolerance. Liquid preparations of iron stain the teeth. They should be administered through a straw, and the mouth rinsed after administration.

The parents of a 14-year-old girl express concerns about the number of hours their daughter spends with her friends. The nurse explains that peer relationships become more important during adolescence because: a. Adolescents dislike their parents. b. Adolescents no longer need parental control. c. They provide adolescents with a feeling of belonging. d. They promote a sense of individuality in adolescents.

ANS: C The peer group serves as a strong support to teenagers, providing them with a sense of belonging and strength and power. During adolescence, the parent-child relationship changes from one of protection-dependency to one of mutual affection and quality. Parents continue to play an important role in personal and health-related decisions. The peer group forms the transitional world between dependence and autonomy.

The school nurse is discussing testicular self-examination with adolescent boys. Why is this important? a. Epididymitis is common during adolescence. b. Asymptomatic sexually transmitted diseases may be present. c. Testicular tumors during adolescence are generally malignant. d. Testicular tumors, although usually benign, are common during adolescence.

ANS: C Tumors of the testes are not common, but when manifested in adolescence, they are generally malignant and demand immediate evaluation. Epididymitis is not common in adolescence. Asymptomatic sexually transmitted disease would not be evident during testicular self-examination. The focus of this examination is on testicular cancer. Testicular tumors are most commonly malignant.

When assessing a child for possible congenital heart defects (CHDs), where should the nurse measure blood pressure? a. The right arm b. The left arm c. All four extremities d. Both arms while the child is crying

ANS: C When a CHD is suspected, the blood pressure should be measured in all four extremities while the child is quiet. Discrepancies between upper and lower extremities may indicate cardiac disease. Blood pressure measurements for upper and lower extremities are compared during an assessment for CHDs. Blood pressure measurements when the child is crying are likely to be elevated; thus the readings will be inaccurate.

The nurse is caring for a 10-month-old infant with respiratory syncytial virus (RSV) bronchiolitis. Which interventions should be included in the child's care (Select all that apply)? a. Administer antibiotics. b. Administer cough syrup. c. Encourage infant to drink 8 ounces of formula every 4 hours. d. Institute cluster care to encourage adequate rest. e. Place on noninvasive oxygen monitoring.

ANS: C, D, E Hydration is important in children with RSV bronchiolitis to loosen secretions and prevent shock. Clustering of care promotes periods of rest. The use of noninvasive oxygen monitoring is recommended.

A nurse is conducting discharge teaching to parents about the care of their infant after cardiac surgery. The nurse instructs the parents to notify the physician if what conditions occur (Select all that apply)? a. Respiratory rate of 36 at rest b. Appetite slowly increasing c. Temperature above 37.7° C (100° F) d. New, frequent coughing e. Turning blue or bluer than normal

ANS: C, D, E The parents should be instructed to notify the physician after their infant's cardiac surgery for a temperature above 37.7° C (100° F); new, frequent coughing; and any episodes of the infant turning blue or bluer than normal. A respiratory rate of 36 at rest for an infant is within normal expectations, and it is expected that the appetite will increase slowly.

The nurse is conducting discharge teaching about signs and symptoms of heart failure to parents of an infant with a repaired tetralogy of Fallot. Which signs and symptoms should the nurse include (Select all that apply)? a. Warm flushed extremities b. Weight loss c. Decreased urinary output d. Sweating (inappropriate) e. Fatigue

ANS: C, D, E The signs and symptoms of heart failure include decreased urinary output, sweating, and fatigue. Other signs include pale, cool extremities, not warm and flushed, and weight gain, not weight loss.

Which clinical manifestations would the nurse expect to see as shock progresses in a child and becomes decompensated shock (Select all that apply)? a. Thirst and diminished urinary output b. Irritability and apprehension c. Cool extremities and decreased skin turgor d. Confusion and somnolence e. Normal blood pressure and narrowing pulse pressure f. Tachypnea and poor capillary refill time

ANS: C, D, F Cool extremities, decreased skin turgor, confusion, somnolence, tachypnea, and poor capillary refill time are beginning signs of decompensated shock.

Nursing interventions for the child after a cardiac catheterization include which of the following (Select all that apply)? a. Allow ambulation as tolerated. b. Monitor vital signs every 2 hours. c. Assess the affected extremity for temperature and color. d. Check pulses above the catheterization site for equality and symmetry. e. Remove pressure dressing after 4 hours. f. Maintain a patent peripheral intravenous catheter until discharge.

ANS: C, F The extremity that was used for access for the cardiac catheterization must be checked for temperature and color. Coolness and blanching may indicate arterial occlusion. The child should have a patent peripheral intravenous line to ensure adequate hydration. Allowing ambulation, monitoring vital signs every 2 hours, checking pulses, and removing the pressure dressing after 4 hours are interventions that do not apply to a child after a cardiac catheterization.

Cystic fibrosis (CF) is suspected in a toddler. Which test is essential in establishing this diagnosis? a. Bronchoscopy c. Urine creatinine b. Serum calcium d. Sweat chloride test

ANS: D A sweat chloride test result greater than 60 mEq/L is diagnostic of CF. Although bronchoscopy is helpful for identifying bacterial infection in children with CF, it is not diagnostic. Serum calcium is normal in children with CF. Urine creatinine is not diagnostic of CF.

The nurse is guiding parents in selecting a day care facility for their child. When making the selection, it is especially important to consider: a. Structured learning environment. b. Socioeconomic status of children. c. Cultural similarities of children. d. Teachers knowledgeable about development.

ANS: D A teacher knowledgeable about development will structure activities for learning. A structured learning environment is not necessary at this age. Socioeconomic status is not the most important factor in selecting a preschool. Preschool is about expanding experiences with others; cultural similarities are not necessary.

Which clinical manifestation should the nurse expect when a child with sickle cell anemia experiences an acute vaso-occlusive crisis? a. Circulatory collapse b. Cardiomegaly, systolic murmurs c. Hepatomegaly, intrahepatic cholestasis d. Painful swelling of hands and feet, painful joints

ANS: D A vaso-occlusive crisis is characterized by severe pain in the area of involvement. If in the extremities, painful swelling of the hands and feet is seen; if in the abdomen, severe pain resembles that of acute surgical abdomen; and if in the head, stroke and visual disturbances occur. Circulatory collapse results from sequestration crises. Cardiomegaly, systolic murmurs, hepatomegaly, and intrahepatic cholestasis result from chronic vaso-occlusive phenomena.

The nurse is preparing a child for possible alopecia from chemotherapy. Which suggestion should be included in the teaching? a. Explaining to the child that hair usually regrows in 1 year. b. Advising the child to expose the head to sunlight to minimize alopecia. c. Explaining to the child that wearing a hat or scarf is preferable to wearing a wig. d. Explaining to the child that, when hair regrows, it may have a slightly different color or texture.

ANS: D Alopecia is a side effect of certain chemotherapeutic agents. When the hair regrows, it may be of different color or texture. The hair usually grows back within 3 to 6 months after the cessation of treatment. The head should be protected from sunlight to avoid sunburn. Children should choose the head covering that they prefer.

What type of shock is characterized by a hypersensitivity reaction causing massive vasodilation and capillary leaks, which may occur with drug or latex allergy? a. Neurogenic shock c. Hypovolemic shock b. Cardiogenic shock d. Anaphylactic shock

ANS: D Anaphylactic shock results from extreme allergy or hypersensitivity to a foreign substance. Neurogenic shock results from loss of neuronal control, such as the interruption of neuronal transmission that occurs from a spinal cord injury. Cardiogenic shock is decreased cardiac output. Hypovolemic shock is a reduction in the size of the vascular compartment, decreasing blood pressure, and low central venous pressure.

An accurate description of anemia is: a. Increased blood viscosity. b. Depressed hematopoietic system. c. Presence of abnormal hemoglobin. d. Decreased oxygen-carrying capacity of blood.

ANS: D Anemia is a condition in which the number of red blood cells or hemoglobin concentration is reduced below the normal values for age. This results in a decreased oxygen-carrying capacity of blood. Increased blood viscosity is usually a function of too many cells or of dehydration, not of anemia. A depressed hematopoietic system or abnormal hemoglobin can contribute to anemia, but the definition depends on the deceased oxygen-carrying capacity of the blood.

A common clinical manifestation of Hodgkin's disease is: a. Petechiae. b. Bone and joint pain. c. Painful, enlarged lymph nodes. d. Enlarged, firm, nontender lymph nodes.

ANS: D Asymptomatic, enlarged, cervical or supraclavicular lymphadenopathy is the most common presentation of Hodgkin's disease. Petechiae are usually associated with leukemia. Bone and joint pain are not likely in Hodgkin's disease. The enlarged nodes are rarely painful.

By what age would the nurse expect that most children could understand prepositional phrases such as "under," "on top of," "beside," and "in back of"? a. 18 months b. 24 months c. 3 years d. 4 years

ANS: D At 4 years, children can understand directional phrases. Children 18 to 24 months and 3 years of age are too young.

Which characteristic best describes the language of a 3-year-old child? a. Asks meanings of words b. Follows directional commands c. Can describe an object according to its composition d. Talks incessantly, regardless of whether anyone is listening

ANS: D Because of the dramatic vocabulary increase at this age, 3-year-olds are known to talk incessantly, regardless of whether anyone is listening. A 4- to 5-year-old asks lots of questions and can follow simple directional commands. A 6-year-old can describe an object according to its composition.

The nurse is recommending how to prevent iron deficiency anemia in a healthy, term, breastfed infant. What should she or he suggest? a. Iron (ferrous sulfate) drops after age 1 month. b. Iron-fortified commercial formula can be used by ages 4 to 6 months. c. Iron-fortified infant cereal can be introduced at age 2 months. d. Iron-fortified infant cereal can be introduced at approximately 6 months of age.

ANS: D Breast milk supplies inadequate iron for growth and development after age 5 months. Supplementation is necessary at this time. Iron supplementation or the introduction of solid foods in a breastfed baby is not indicated. Introducing iron-fortified infant cereal at 2 months should be done only if the mother is choosing to discontinue breastfeeding.

A common, serious complication of rheumatic fever is: a. Seizures. c. Pulmonary hypertension. b. Cardiac arrhythmias. d. Cardiac valve damage.

ANS: D Cardiac valve damage is the most significant complication of rheumatic fever. Seizures, cardiac arrhythmias, and pulmonary hypertension are not common complications of rheumatic fever.

Skin testing for tuberculosis (the Mantoux test) is recommended: a. Every year for all children older than 2 years. b. Every year for all children older than 10 years. c. Every 2 years for all children starting at age 1 year. d. Periodically for children who reside in high-prevalence regions.

ANS: D Children who reside in high prevalence regions for tuberculosis should be tested every 2 to 3 years. Annual testing is not necessary. Testing is not necessary unless exposure is likely or an underlying medical risk factor is present.

By what age should concerns about pubertal delay be considered in boys? a. 12 to 12.5 years c. 13 to 13.5 years b. 12.5 to 13 years d. 13.5 to 14 years

ANS: D Concerns about pubertal delay should be considered for boys who exhibit no enlargement of the testes or scrotal changes by 13.5 to 14 years of age. Ages younger than 13.5 years are too young for initial concern.

Which is probably the most important criterion on which to base the decision to report suspected child abuse? a. Inappropriate parental concern for the degree of injury b. Absence of parents for questioning about child's injuries c. Inappropriate response of child d. Incompatibility between the history and injury observed

ANS: D Conflicting stories about the "accident" are the most indicative red flags of abuse. Inappropriate response of caregiver or child may be present, but is subjective. Parents should be questioned at some point during the investigation.

The parents of a young child with congestive heart failure tell the nurse that they are "nervous" about giving digoxin. The nurse's response should be based on knowing that: a. It is a safe, frequently used drug. b. It is difficult to either overmedicate or undermedicate with digoxin. c. Parents lack the expertise necessary to administer digoxin. d. Parents must learn specific, important guidelines for administration of digoxin.

ANS: D Digoxin has a narrow therapeutic range. The margin of safety between therapeutic, toxic, and lethal doses is very small. Specific guidelines are available for parents to learn how to administer the drug safely and monitor for side effects. Digoxin is a frequently used drug, but it has a narrow therapeutic range. Very small amounts of the liquid are given to infants, which makes it easy to overmedicate or undermedicate. Parents may lack the necessary expertise to administer the drug at first, but with discharge preparation they should be prepared to administer the drug safely.

Pancreatic enzymes are administered to the child with cystic fibrosis. Nursing considerations should include: a. Do not administer pancreatic enzymes if the child is receiving antibiotics. b. Decrease dose of pancreatic enzymes if the child is having frequent, bulky stools. c. Administer pancreatic enzymes between meals if at all possible. d. Pancreatic enzymes can be swallowed whole or sprinkled on a small amount of food taken at the beginning of a meal.

ANS: D Enzymes may be administered in a small amount of cereal or fruit or swallowed whole at the beginning of a meal, not between meals. Pancreatic enzymes are not contraindicated with antibiotics. The dose of enzymes should be increased if the child is having frequent, bulky stools.

Which vitamin supplements are necessary for children with cystic fibrosis? a. Vitamin C and calcium c. Magnesium b. Vitamins B6 and B12 d. Vitamins A, D, E, and K

ANS: D Fat-soluble vitamins are poorly absorbed because of deficient pancreatic enzymes in children with cystic fibrosis; therefore, supplements are necessary. Vitamin C and calcium are not fat soluble. Vitamins B6 and B12 are not fat-soluble vitamins. Magnesium is a mineral, not a vitamin.

A child with leukemia is receiving triple intrathecal chemotherapy consisting of methotrexate, cytarabine, and hydrocortisone. The purpose of this is to prevent: a. Infection. b. Brain tumor c. Drug side effects. d. Central nervous system (CNS) disease.

ANS: D For certain children, CNS prophylactic therapy is indicated. This drug regimen is used to prevent CNS leukemia. This regimen does not prevent infection or drug side effects. If the child has a brain tumor in addition to leukemia, additional therapy would be indicated.

Which is the causative agent of scarlet fever? a. Enteroviruses b. Corynebacterium organisms c. Scarlet fever virus d. Group A b-hemolytic streptococci (GABHS)

ANS: D GABHS infection causes scarlet fever. Enteroviruses do not cause the same complications. Corynebacterium organisms cause diphtheria. Scarlet fever is not caused by a virus.

When discussing hyperlipidemia with a group of adolescents, the nurse should explain that high levels of what substance are thought to protect against cardiovascular disease? a. Cholesterol c. Low-density lipoproteins (LDLs) b. Triglycerides d. High-density lipoproteins (HDLs).

ANS: D HDLs contain very low concentrations of triglycerides, relatively little cholesterol, and high levels of proteins. It is thought that HDLs protect against cardiovascular disease. Cholesterol, triglycerides, and LDLs do not protect against cardiovascular disease.

When caring for the child with Kawasaki disease, the nurse should understand that: a. The child's fever is usually responsive to antibiotics within 48 hours. b. The principal area of involvement is the joints. c. Aspirin is contraindicated. d. Therapeutic management includes administration of gamma globulin and aspirin.

ANS: D High-dose intravenous gamma globulin and aspirin therapy are indicated to reduce the incidence of coronary artery abnormalities when given within the first 10 days of the illness. The fever of Kawasaki disease is unresponsive to antibiotics and antipyretics. Involvement of mucous membranes and conjunctiva, changes in the extremities, and cardiac involvement are seen.

Which statement is characteristic of acute otitis media (AOM)? a. The etiology is unknown. b. Permanent hearing loss often results. c. It can be treated by intramuscular antibiotics. d. It is treated with a broad range of antibiotics.

ANS: D Historically AOM has been treated with a range of antibiotics, and it is the most common disorder treated with antibiotics in the ambulatory setting. The etiology of AOM may be bacterial, such as Streptococcus pneumoniae, Haemophilus influenzae, and Moraxella catarrhalis, or a viral agent. Recent concerns about drug-resistant organisms have caused authorities to recommend judicious use of antibiotics and that antibiotics are not required for initial treatment. Permanent hearing loss is not a frequent cause of properly treated AOM. Intramuscular antibiotics are not necessary. Oral amoxicillin is the treatment of choice.

An acquired hemorrhagic disorder that is characterized by excessive destruction of platelets is: a. Aplastic anemia. b. Thalassemia major. c. Disseminated intravascular coagulation. d. Idiopathic thrombocytopenic purpura.

ANS: D Idiopathic thrombocytopenic purpura is an acquired hemorrhagic disorder characterized by an excessive destruction of platelets, discolorations caused by petechiae beneath the skin, and a normal bone marrow. Aplastic anemia refers to a bone marrow failure condition in which the formed elements of the blood are simultaneously depressed. Thalassemia major is a group of blood disorders characterized by deficiency in the production rate of specific hemoglobin globin chains. Disseminated intravascular coagulation is characterized by diffuse fibrin deposition in the microvasculature, consumption of coagulation factors, and endogenous generation of thrombin and plasma.

The parent of an infant with nasopharyngitis should be instructed to notify the health care professional if the infant: a. Becomes fussy. c. Has a fever over 99° F. b. Has a cough. d. Shows signs of an earache.

ANS: D If an infant with nasopharyngitis has a fever over 101° F, there is early evidence of respiratory complications. Irritability and a slight fever are common in an infant with a viral illness. Cough can be a sign of nasopharyngitis.

The nurse is caring for a school-age girl who has had a cardiac catheterization. The child tells the nurse that her bandage is "too wet." The nurse finds the bandage and bed soaked with blood. The most appropriate initial nursing action is to: a. Notify the physician. b. Apply a new bandage with more pressure. c. Place the child in the Trendelenburg position. d. Apply direct pressure above the catheterization site.

ANS: D If bleeding occurs, direct continuous pressure is applied 2.5 cm (1 inch) above the percutaneous skin site to localize pressure over the vessel puncture. Notifying the physician and applying a new bandage with more pressure can be done after pressure is applied. The nurse can have someone else notify the physician while the pressure is being maintained. The Trendelenburg position would not be helpful; it would increase the drainage from the lower extremities.

What describes the cognitive abilities of school-age children? a. Have developed the ability to reason abstractly b. Become capable of scientific reasoning and formal logic c. Progress from making judgments based on what they reason to making judgments based on what they see d. Have the ability to classify, group and sort, and hold a concept in their minds while making decisions based on that concept

ANS: D In Piaget's stage of concrete operations, children have the ability to group and sort and make conceptual decisions. Children cannot reason abstractly until late adolescence. Scientific reasoning and formal logic are skills of adolescents. Making judgments on what the child sees versus what he or she reasons is not a developmental skill.

In girls, the initial indication of puberty is: a.Menarche. c. Growth of pubic hair. b. Growth spurt. d. Breast development.

ANS: D In most girls, the initial indication of puberty is the appearance of breast buds, an event known as the larche. The usual sequence of secondary sexual characteristic development in girls is breast changes, rapid increase in height and weight, growth of pubic hair, appearance of axillary hair, menstruation, and abrupt deceleration of linear growth.

Seventy-two hours after cardiac surgery, a young child has a temperature of 37.7° C (101° F). The nurse should: a. Keep the child warm with blankets. b. Apply a hypothermia blanket. c. Record the temperature on nurses' notes. d. Report findings to physician.

ANS: D In the first 24 to 48 hours after surgery, the body temperature may increase to 37.7° C (100° F) as part of the inflammatory response to tissue trauma. If the temperature is higher or an elevated temperature continues after this period, it is most likely a sign of an infection and immediate investigation is indicated. Blankets should be removed from the child to keep the temperature from increasing. A hypothermia blanket is not indicated for this level of temperature. The temperature should be recorded, but the physician must be notified for evaluation.

Which statement is most descriptive of bulimia during adolescence? a. Strong sense of control over eating behavior b. Feelings of elation after the binge-purge cycle c. Profound lack of awareness that the eating pattern is abnormal d. Weight that can be normal, slightly above normal, or below normal

ANS: D Individuals with bulimia are of normal weight or more commonly slightly above normal weight. Those who also restrict their intake can become severely underweight. Behavior related to this eating disorder is secretive, frenzied, and out of control. These cycles are followed by self-deprecating thoughts and a depressed mood. These young women are keenly aware that this eating pattern is abnormal.

Which information should the nurse teach workers at a day care center about respiratory syncytial virus (RSV)? a. RSV is transmitted through particles in the air. b. RSV can live on skin or paper for up to a few seconds after contact. c. RSV can survive on nonporous surfaces for about 60 minutes. d. Frequent hand washing can decrease the spread of the virus.

ANS: D Meticulous hand washing can decrease the spread of organisms. RSV infection is not airborne. It is acquired mainly through contact with contaminated surfaces. RSV can live on skin or paper for up to 1 hour and on cribs and other nonporous surfaces for up to 6 hours.

Chronic otitis media with effusion (OME) is differentiated from acute otitis media (AOM) because it is usually characterized by: a. Fever as high as 40° C (104° F). c. Nausea and vomiting. b. Severe pain in the ear. d. A feeling of fullness in the ear.

ANS: D OME is characterized by an immobile or orange-discolored tympanic membrane and nonspecific complaints and does not cause severe pain. Fever and severe pain may be signs of AOM. Nausea and vomiting are associated with otitis media.

What is characteristic of dishonest behavior in children ages 8 to 10 years? a. Cheating during games is now more common. b. Lying results from the inability to distinguish between fact and fantasy. c. They may steal because their sense of property rights is limited. d. They may lie to meet expectations set by others that they have been unable to attain.

ANS: D Older school-age children may lie to meet expectations set by others to which they have been unable to measure up. Cheating usually becomes less frequent as the child matures. In this age group, children are able to distinguish between fact and fantasy. Young children may lack a sense of property rights; older children may steal to supplement an inadequate allowance, or it may be an indication of serious problems.

Parents have understood teaching about prevention of childhood otitis media if they make which statement? a. "We will only prop the bottle during the daytime feedings." b. "Breastfeeding will be discontinued after 4 months of age." c. "We will place the child flat right after feedings." d. "We will be sure to keep immunizations up to date."

ANS: D Parents have understood the teaching about preventing childhood otitis media if they respond they will keep childhood immunizations up to date. The child should be maintained upright during feedings and after. Otitis media can be prevented by exclusively breastfeeding until at least 6 months of age. Propping bottles is discouraged to avoid pooling of milk while the child is in the supine position.

The role of the peer group in the life of school-age children is that it: a. Gives them an opportunity to learn dominance and hostility. b. Allows them to remain dependent on their parents for a longer time. c. Decreases their need to learn appropriate sex roles. d. Provides them with security as they gain independence from their parents.

ANS: D Peer-group identification is an important factor in gaining independence from parents. Through peer relationships, children learn ways to deal with dominance and hostility. They also learn how to relate to people in positions of leadership and authority and explore ideas and the physical environment. Peer-group identification helps in gaining independence rather than remaining dependent. A child's concept of appropriate sex roles is influenced by relationships with peers.

José is a 4-year-old child scheduled for a cardiac catheterization. Preoperative teaching should be: a. Directed at his parents because he is too young to understand. b. Detailed in regard to the actual procedures so he will know what to expect. c. Done several days before the procedure so that he will be prepared. d. Adapted to his level of development so that he can understand.

ANS: D Preoperative teaching should always be directed at the child's stage of development. The caregivers also benefit from the same explanations. The parents may ask additional questions, which should be answered, but the child needs to receive the information based on developmental level. This age group does not understand in-depth descriptions. Preschoolers should be prepared close to the time of the cardiac catheterization.

When caring for an infant with an upper respiratory tract infection and elevated temperature, an appropriate nursing intervention is to: a. Give tepid water baths to reduce fever. b. Encourage food intake to maintain caloric needs. c. Have child wear heavy clothing to prevent chilling. d. Give small amounts of favorite fluids frequently to prevent dehydration.

ANS: D Preventing dehydration by small frequent feedings is an important intervention in the febrile child. Tepid water baths may induce shivering, which raises temperature. Food should not be forced; it may result in the child vomiting. The febrile child should be dressed in light, loose clothing.

When planning care for adolescents, the nurse should: a. Teach parents first, and they, in turn, will teach the teenager. b. Provide information for their long-term health needs because teenagers respond best to long-range planning. c. Maintain the parents' role by providing explanations for treatment and procedures to the parents only. d. Give information privately to adolescents about how they can manage the specific problems that they identify.

ANS: D Problems that teenagers identify and are interested in are typically the problems that they are the most willing to address. Confidentiality is important to adolescents. Adolescents prefer to confer privately (without parents) with the nurse and health care provider. Teenagers are socially and cognitively at the developmental stage where the health care provider can teach them. The nurse must keep in mind that teenagers are more interested in immediate health care needs than in long-term needs.

For what reason might a newborn infant with a cardiac defect, such as coarctation of the aorta, that results in a right-to-left shunt receive prostaglandin E1? a.To decrease inflammation c. To decrease respirations b. To control pain d. To improve oxygenation

ANS: D Prostaglandin E1 is given to infants with a right-to-left shunt to keep the ductus arteriosus patent. This will improve oxygenation and increase pulmonary blood flow.

Which common childhood communicable disease may cause severe defects in the fetus when it occurs in its congenital form? a. Erythema infectiosum b. Roseola c. Rubeola d. Rubella

ANS: D Rubella causes teratogenic effects on the fetus. There is a low risk of fetal death to those in contact with children affected with fifth disease. Roseola and rubeola are not dangerous to the fetus.

Which comment is most developmentally typical of a 7-year-old boy? a. "I am a Power Ranger, so don't make me angry." b. "I don't know whether I like Mary or Joan better." c. "My mom is my favorite person in the world." d. "Jimmy is my best friend."

ANS: D School-age children form friendships with peers of the same sex, those who live nearby, and other children who have toys that they enjoy sharing. Magical thinking is developmentally appropriate for the preschooler. Opposite-sex friendships are not typical for the 7-year-old child. Seven-year-old children socialize with their peers, not their parents.

Several complications can occur when a child receives a blood transfusion. An immediate sign or symptom of an air embolus is: a. Chills and shaking. c. Irregular heart rate. b. Nausea and vomiting. d. Sudden difficulty in breathing.

ANS: D Signs of air embolism are sudden difficulty breathing, sharp pain in the chest, and apprehension. Air emboli should be avoided by carefully flushing all tubing of air before connecting to the patient. Chills, shaking, nausea, and vomiting are associated with hemolytic reactions. Irregular heart rate is associated with electrolyte disturbances and hypothermia.

A child has had cold symptoms for more than 2 weeks, a headache, nasal congestion with purulent nasal drainage, facial tenderness, and a cough that increases during sleep. The nurse recognizes that these symptoms are characteristic of which respiratory condition? a. Allergic rhinitis c. Asthma b. Bronchitis d. Sinusitis

ANS: D Sinusitis is characterized by signs and symptoms of a cold that do not improve after 14 days, a low-grade fever, nasal congestion and purulent nasal discharge, headache, tenderness, a feeling of fullness over the affected sinuses, halitosis, and a cough that increases when the child is lying down. The classic symptoms of allergic rhinitis are watery rhinorrhea; itchy nose, eyes, ears, and palate; and sneezing. Symptoms occur as long as the child is exposed to the allergen. Bronchitis is characterized by a gradual onset of rhinitis and a cough that is initially nonproductive but may change to a loose cough. The manifestations of asthma may vary, with wheezing being a classic sign. The symptoms presented in the question do not suggest asthma.

An adolescent teen has bulimia. Which assessment finding should the nurse expect? a. Diarrhea c. Cold intolerance b. Amenorrhea d. Erosion of tooth enamel

ANS: D Some of the signs of bulimia include erosion of tooth enamel, increased dental caries from vomited gastric acid, throat complaints, fluid and electrolyte disturbances, and abdominal complaints from laxative abuse. Diarrhea is not a result of the vomiting. It may occur in patients with bulimia who also abuse laxatives. Amenorrhea and cold intolerance are characteristics of anorexia nervosa, which some bulimics have. These symptoms are related to the extreme low weight.

The most common causative agent of bacterial endocarditis is: a. Staphylococcus albus. c. Staphylococcus albicans. b. Streptococcus hemolyticus. d. Streptococcus viridans.

ANS: D Staphylococcus viridans is the most common causative agent in bacterial (infective) endocarditis. Staphylococcus albus, Streptococcus hemolyticus, and Staphylococcus albicans are not common causative agents.

Which symptoms should the nurse expect to observe during the physical assessment of an adolescent girl with severe weight loss and disrupted metabolism associated with anorexia nervosa? a. Dysmenorrhea and oliguria b. Tachycardia and tachypnea c. Heat intolerance and increased blood pressure d. Lowered body temperature and brittle nails

ANS: D Symptoms of anorexia nervosa include lower body temperature, severe weight loss, decreased blood pressure, dry skin, brittle nails, altered metabolic activity, and presence of lanugo hair. Amenorrhea, rather than dysmenorrhea, and cold intolerance are manifestations of anorexia nervosa. Bradycardia, rather than tachycardia, may be present.

A school nurse is teaching dental health practices to a group of sixth-grade children. How often should the nurse recommend the children brush their teeth? a. Twice a day b. Three times a day c. After meals d. After meals and snacks, and at bedtime

ANS: D Teeth should be brushed after meals, after snacks, and at bedtime. Children who brush their teeth frequently and become accustomed to the feel of a clean mouth at an early age usually maintain the habit throughout life. Twice a day, three times a day, or only after meals would not be often enough.

Why do infants and young children quickly have respiratory distress in acute and chronic alterations of the respiratory system? a. They have a widened, shorter airway. b. There is a defect in their sucking ability. c. The gag reflex increases mucus production. d. Mucus and edema obstruct small airways.

ANS: D The airway in infants and young children is narrower, not wider, and respiratory distress can occur quickly because mucus and edema can cause obstruction to their small airways. Sucking is not necessarily related to problems with the airway. The gag reflex is necessary to prevent aspiration. It does not produce mucus.

Steve, 14 years old, mentions that he now has to use deodorant but never had to before. The nurse's response should be based on knowledge that: a. Eccrine sweat glands in the axillae become fully functional during puberty. b. Sebaceous glands become extremely active during puberty. c. New deposits of fatty tissue insulate the body and cause increased sweat production. d. Apocrine sweat glands reach secretory capacity during puberty.

ANS: D The apocrine sweat glands, nonfunctional in children, reach secretory capacity during puberty. They secrete a thick substance as a result of emotional stimulation that, when acted on by surface bacteria, becomes highly odoriferous. They are limited in distribution and grow in conjunction with hair follicles in the axillae, genital and anal areas, and other areas. Eccrine sweat glands are present almost everywhere on the skin and become fully functional and respond to emotional and thermal stimulation. Sebaceous glands become extremely active at this time, especially those on the genitals and the "flush" areas of the body, such as face, neck, shoulders, upper back, and chest. This increased activity is important in the development of acne. New deposits of fatty tissue insulate the body and cause increased sweat production, but this is not the etiology of apocrine sweat gland activity.

Which postoperative intervention should be questioned for a child after a cardiac catheterization? a. Continue intravenous (IV) fluids until the infant is tolerating oral fluids. b. Check the dressing for bleeding. c. Assess peripheral circulation on the affected extremity. d. Keep the affected leg flexed and elevated.

ANS: D The child should be positioned with the affected leg straight for 4 to 6 hours after the procedure. IV fluid administration continues until the child is taking and retaining adequate amounts of oral fluids. The insertion site dressing should be observed frequently for bleeding. The nurse should also look under the child to check for pooled blood. Peripheral perfusion is monitored after catheterization. Distal pulses should be palpable, although they may be weaker than in the contralateral extremity.

What is an expected assessment finding in a child with coarctation of the aorta? a. Orthostatic hypotension b. Systolic hypertension in the lower extremities c. Blood pressure higher on the left side of the body d. Disparity in blood pressure between the upper and lower extremities

ANS: D The classic finding in children with coarctation of the aorta is a disparity in pulses and blood pressures between the upper and lower extremities. Orthostatic hypotension is not present with coarctation of the aorta. Systolic hypertension may be detected in the upper extremities. The left arm may not accurately reflect systolic hypertension because the left subclavian artery can be involved in the coarctation.

Myelosuppression associated with chemotherapeutic agents or some malignancies such as leukemia can cause bleeding tendencies because of a/an: a. Decrease in leukocytes. c. Vitamin C deficiency. b. Increase in lymphocytes. d. Decrease in blood platelets.

ANS: D The decrease in blood platelets secondary to the myelosuppression of chemotherapy can cause an increase in bleeding. The child and family should be alerted to avoid risk of injury. Decrease in leukocytes, increase in lymphocytes, and vitamin C deficiency would not affect bleeding tendencies.

Surgical closure of the ductus arteriosus would: a. Stop the loss of unoxygenated blood to the systemic circulation. b. Decrease the edema in legs and feet. c. Increase the oxygenation of blood. d. Prevent the return of oxygenated blood to the lungs.

ANS: D The ductus arteriosus allows blood to flow from the higher-pressure aorta to the lower-pressure pulmonary artery, causing a right-to-left shunt. If this is surgically closed, no additional oxygenated blood (from the aorta) will return to the lungs through the pulmonary artery. The aorta carries oxygenated blood to the systemic circulation. Because of the higher pressure in the aorta, blood is shunted into the pulmonary artery and the pulmonary circulation. Edema in the legs and feet is usually a sign of heart failure. This repair would not directly affect the edema. Increasing the oxygenation of blood would not interfere with the return of oxygenated blood to the lungs.

Anorexia nervosa may best be described as: a. Occurring most frequently in adolescent males. b. Occurring most frequently in adolescents from lower socioeconomic groups. c. Resulting from a posterior pituitary disorder. d. Resulting in severe weight loss in the absence of obvious physical causes.

ANS: D The etiology of anorexia remains unclear, but a distinct psychologic component is present. The diagnosis is based primarily on psychologic and behavioral criteria. Anorexia nervosa is observed more commonly in adolescent girls and young women. It does not occur most frequently in adolescents from a lower socioeconomic group. In reality, anorexic adolescents are often from families of means who have high parental expectations for achievement. Anorexia is a psychiatric disorder.

The mother of a toddler yells to the nurse, "Help! He is choking to death on his food." The nurse determines that lifesaving measures are necessary based on: a. Gagging. c. Pulse over 100 beats/min. b. Coughing. d. Inability to speak.

ANS: D The inability to speak indicates a foreign-body airway obstruction of the larynx. Abdominal thrusts are needed for treatment of the choking child. Gagging indicates irritation at the back of the throat, not obstruction. Coughing does not indicate a complete airway obstruction. Tachycardia may be present for many reasons.

The most common cause of death in the adolescent age-group involves: a. Drownings. c.Drug overdoses. b. Firearms. d. Motor vehicles.

ANS: D The leading cause of all adolescent deaths in the United States is motor vehicle accidents. Drownings, firearms, and drug overdoses are major concerns in adolescence but do not cause the majority of deaths.

Which behavior is not normally demonstrated in the 8-year-old child? a. Understands that his or her point of view is not the only one b. Enjoys telling riddles and silly jokes c. Understands that pouring liquid from a small to a large container does not change the amount d. Engages in fantasy and magical thinking

ANS: D The preschool child engages in fantasy and magical thinking. The school-age child moves away from this type of thinking and becomes more skeptical and logical. Belief in Santa Claus or the Easter Bunny ends in this period of development. School-age children enter the stage of concrete operations. They learn that their point of view is not the only one. The school-age child has a sense of humor. The child's increased language mastery and increased logic allow for appreciation of plays on words, jokes, and incongruities. The school-age child understands that properties of objects do not change when their order, form, or appearance does.

An 8-year-old girl is receiving a blood transfusion when the nurse notes that she has developed precordial pain, dyspnea, distended neck veins, slight cyanosis, and a dry cough. These manifestations are most suggestive of: a. Air embolism. c. Hemolytic reaction. b. Allergic reaction. d. Circulatory overload.

ANS: D The signs of circulatory overload include distended neck veins, hypertension, crackles, dry cough, cyanosis, and precordial pain. Signs of air embolism are sudden difficulty breathing, sharp pain in the chest, and apprehension. Allergic reactions are manifested by urticaria, pruritus, flushing, asthmatic wheezing, and laryngeal edema. Hemolytic reactions are characterized by chills, shaking, fever, pain at infusion site, nausea, vomiting, tightness in chest, flank pain, red or black urine, and progressive signs of shock and renal failure.

Which immunization should not be given to a child receiving chemotherapy for cancer? a. Tetanus vaccine c. Diphtheria, pertussis, tetanus (DPT) b. Inactivated poliovirus vaccine d. Measles, rubella, mumps

ANS: D The vaccine used for measles, mumps, and rubella is a live virus and can result in an overwhelming infection. Tetanus vaccine, inactivated poliovirus vaccine, and DPT are not live virus vaccines.

An adolescent boy tells the nurse that he has recently had homosexual feelings. The nurse's response should be based on knowledge that: a. This indicates that the adolescent is homosexual. b. This indicates that the adolescent will become homosexual as an adult. c. The adolescent should be referred for psychotherapy. d. The adolescent should be encouraged to share his feelings and experiences.

ANS: D These adolescents are at increased risk for health-damaging behaviors, not because of the sexual behavior itself, but because of society's reaction to the behavior. The nurse's first priority is to give the young man permission to discuss his feelings about this topic, knowing that the nurse will maintain confidentially, appreciate his feelings, and remain sensitive to his need to talk it. In recent studies among self-identified gay, lesbian, and bisexual adolescents, many of the adolescents report changing their self-labels one or more times during their adolescence.

The father of 12-year-old Ryan tells the nurse that he is concerned about his son getting "fat." Ryan's body mass index for age is at the 60th percentile. The most appropriate nursing action is to: a. Reassure the father that Ryan is not "fat." b. Reassure the father that Ryan is just a growing child. c. Suggest a low-calorie, low-fat diet. d. Explain that this is typical of the growth pattern of boys at this age.

ANS: D This is a characteristic pattern of growth in preadolescent boys, in which the growth in height has slowed in preparation for the pubertal growth spurt but weight is still gained. This should be reviewed with both the father and Ryan, and a plan should be developed to maintain physical exercise and a balanced diet. Saying that Ryan is not "fat" is false reassurance. His weight is high for his height. Ryan needs to maintain his physical activity. The father is concerned; an explanation is required. A nutritional diet with physical activity should be sufficient to maintain his balance.

In terms of fine motor development, what could the 3-year-old child be expected to do? a. Tie shoelaces. b. Use scissors or a pencil very well. c. Draw a person with seven to nine parts. d. Copy (draw) a circle.

ANS: D Three-year-olds are able to accomplish the fine motor skill of drawing a circle. Tying shoelaces, using scissors or a pencil very well, and drawing a person with multiple parts are fine motor skills of 5-year-old children.

Which accomplishment would the nurse expect of a healthy 3-year-old child? a. Jump rope b. Ride a two-wheel bicycle c. Skip on alternate feet d. Balance on one foot for a few seconds

ANS: D Three-year-olds are able to accomplish the gross motor skill of balancing on one foot. Jumping rope, riding a two-wheel bike, and skipping on alternate feet are gross motor skills of 5-year-old children.

Which medication may be given to high risk children after exposure to chickenpox to prevent varicella? a. Acyclovir b. Vitamin A c. Diphenhydramine hydrochloride d. Varicella zoster immune globulin (VZIG)

ANS: D VZIG is given to high risk children to help prevent the development of chickenpox. Immune globulin intravenous may also be recommended. Acyclovir is given to immunocompromised children to reduce the severity of symptoms. Vitamin A reduces morbidity and mortality associated with the measles. The antihistamine diphenhydramine is administered to reduce the itching associated with chickenpox.

The weight loss of anorexia nervosa is often triggered by: a. Sexual abuse. c. Independence from family. b. School failure. d. Traumatic interpersonal conflict.

ANS: D Weight loss may be triggered by a typical adolescent crisis such as the onset of menstruation or a traumatic interpersonal incident; situations of severe family stress such as parental separation or divorce; or circumstances in which the young person lacks personal control, such as being teased, changing schools, or entering college. There may in fact be a history of sexual abuse; however, this is not the trigger. These adolescents are often overachievers who are successful in school, not failures in school. The adolescent is most often enmeshed with his or her family.

The school nurse has been asked to begin teaching sex education in the 5th grade. The nurse should recognize that: a. Children in 5th grade are too young for sex education. b. Children should be discouraged from asking too many questions. c. Correct terminology should be reserved for children who are older. d. Sex can be presented as a normal part of growth and development.

ANS: D When sex information is presented to school-age children, sex should be treated as a normal part of growth and development. Fifth graders are usually 10 to 11 years old. This age is not too young to speak about physiologic changes in their bodies. They should be encouraged to ask questions. Preadolescents need precise and concrete information.


संबंधित स्टडी सेट्स

COMPTIA A+ 220-1002: MICROSOFT COMMAND LINE TOOLS

View Set

Fundamentals success Medication administration questions

View Set

Exam 2 Bio Drugs and society chapter 4

View Set

Ancient Philosophy Final: Dr. Sickler

View Set

National Treasury Employee Union v. Von Raab

View Set

Macroeconomics Chapter 16 Homework

View Set

Macroeconomics Ch.30 (Labor Market & Unemployment)

View Set